Está en la página 1de 250
INSTRUCTOR’S SOLUTIONS MANUAL INTRODUCTION to ELECTRODYNAMICS Third Edition David J. Griffiths Chapter 1 Chapter 2 Chapter 3 Chapter 4 ‘Chapter 5 Chapter 6 Chapter 7 Chapter 8 Chapter 9 Chapter 10 Chapter 11 Chapter 12 TABLE OF CONTENTS Vector Analysis Electrostatics Special Techniques Electrostatic Fields in Matter Magnetostatics Magnetostatic Fields in Matter Electrodynamics Conservation Laws Electromagnetic Waves Potentials and Fields Radiation Electrodynamics and Relativity 73 89) 113 125 Chapter 1 Vector Analysis Problem 1.1 (2) From the diagram, |B + C|cos6 = [B| cos + |C}cosd,. Multiply by |A\. AIIB + C]cosés = |Al|B| cos: + |Al|C] cos 6, Clsines So: A(B+C) = A-B+A.C. (Dot product is distributive.) Similarly: |B + C|sin y = [B|sin 0) + {C|sin@,. Mulitply by |Al a. JAIIB + C|sin dy 8 = [Al)B| sin, f+ [Al|C|sin & 8 (Bisine, If fis the unit vector pointing out of the page, it follows that, Ax(B +) = (AxB) +(AXC). (Cross product is distributive.) (Bicone (Clomes (b) For the general case, see G. E. Hay’s Vector and Tensor Analysis, Chapter 1, Section 7 (dot product) and Section 8 (cross product). Problem 1.2 The triple cross-produet is not in general associative, For example, i suppose A = B and C is perpendicular to A, as in the diagram. Then (BXC) points out-of-the-page, and Ax(BxC) points down, A=B and has magnitude ABC. But (AXB) = 0, so (AxB)xC = 04 Ax(BxC), BxC jAax(BxC) Problem 13 = A=+1R+19- 18; A= V3; B=1k+19 +14; B= V3. AB = 4141-121 = AB cosd = V3VEc080 = con® =} Jy [6 = cos"! (}) = 70.524 K Problem 14 ‘The eross-produet of any two vectors in the plane will give a vector perpendicular to the plane. For example, wwe might pick the base (A) and the left side (B). A 18429 +08; B=-12409-+38. 2 CHAPTER !, VECTOR ANALYSIS This has the right direction, but the wrong magnitude, ‘To make a unit vector out of it; simply divide by its length: |AxB) = VBFOF4 = 7, Problem 15 & 5 2 | Ax(BxC) = | A: Ay As | | (ByC.-B.C,) (BsCe-BeC,) (BeC, ~ B,C.) | = 8[Ay(BeCy ~ ByCx) ~ As(BsCz - BeCz)] +90) + 81) (I just check the x-component; the others go the same way.) = 8(4yBzC, — AyByCz ~ A:B,C, + ArBzC.) + 9() +2). B(A-C) - C(A-B) = [Bz(AzCz + AyCy + AsC.) ~ Cz(ArBe + AyBy + ArB.)|+ (H+ ()% (AyBsCy + A,ByCz ~ AyByCz ~ AzB,Ce) +9() +2(). They agree. Problem 1.6 Ax(BXC)4Bx(CxA)+Cx(AXB) = B(A-C)~C(A-B)+C(A-B)-A(C-B)+A(B-C)-B(C-A) = 0 §0: Ax(BXC) ~ (AxB)xC = ~Bx(CxA) = A(B-C) ~ C(A-B) If this is zero, then either A is parallel to C (including the case in which they point in opposite directions, or one is zero), or else B-C = B+A = 0, in which case B is perpendicular to A and C (including the case B = 0). Conclusion: [Ax(BXC) = (AXB)xC <= either A is parallel to C, or B is perpendicular to A and C.] Problem 1.7 a= (48465482) ~ (28489478) = [DR-2H 42 =vivtri =[3] Problem 18 (a) A,B, + ALB, = (cos dy + sin pA,)(cos pBy + sin dB.) + (~sin dA, + cos dA,)(— sin 6B, + cos 6B:) = cos? GAyBy + sin dcos (A,B, + AzB,) + sin? GA,Bz + sin? 34,By — sindcos¢(AyBs + AzBy) + cos GA Bs cos? } + sin? 4)A,B, + (sin? d+ cos? @)AsBs = AyBy + Axe. ¥ (b) He)? + (Gy)? + Ay? = DAA = Dh, (Bhar RisAj) (Bhar Rade) = Dje (DiRy Rie) Aye. ‘This equals A2 + A? + A? provided] DL, Ry Rix ={ A y jst } Moreover, if R is to preserve lengths for all vectors A, then this condition is not only sufficient but also necessary. For suppose A = (1,0,0). Then Bx (Bi Ri Ria) Aye = Bi Ra Rey and this must equa 1 (since we want A, +4/-+A: = 1), Likewise, DL, RigRia = EL, Rig Rea = 1. To check the case j £ k, choose A = (1,1,0). Then we want 2= Zjx (Bi Raju) As = Di Ria + Es Rahs +E; Raa + ¥; RigRa- But we already know that the first two sums are both 1; the third and fourth are equal, so Di Ry: Ry = Ej Ri2Ra = 0, and so on for other tiequal combinations of jv In matrix notation: AR — 1, where iis the transpose of R. 3 Problem 1.9 y | y 2 Looking down the asi: R 2, aw ‘A 120° rotation carries the z axis into the y (= 2) axis, y into z (=), and 2 into z (= 7). So Ae = By= dn A= 4, ool R=(100 o10 Problem 1.10 (a) (No change. | (A, = Ae, Ay = Ay, Ae = As) (b) [A 9 =A, ]in the sense (A, = ~Ay, Ay = As) (6) (AxB) —> (~A)x(—B) = (AB), That is, if C = AxB, [E—)C] No minus sign, in contrast to behavior of an “ordinary” vector, as given by (b). If A and B are pseudovectors, then (A xB) + (A)x(B) = (Ax:B). So the cross-product of two pscudovectors is again a pseudovector. In the cross-product of a vector and a pscudovector, one changes sign, the other doesn’t, and therefore the cross-product is itself a vector. Angular momentum (L = tXp) and torque (N = rxF) are pseudovectors. (@) ABxC) S (-A)((-B)x(-C)) = ~A(BXC), So, fa = A-(BXC), then [a —> may] pecudoscalar changes sign under inversion of coordinates. Problem 111 VS = 2x4 By Ft 4re yy Ar VS = Qayrat + 3a2y?2t G4 srtyte a (OVS = eFsinyln2% + eFcosylnz +e siny(I/2) Problem 112 (a) VA = 10,(2y - 62 — 18) & + (22 ~ 8y +28)9]. VA = O at summit, co 2y — 62 =I 2 . m+ be Day + 84 Jays Dy +84 =0. 22y = 66 —p y= 35 22-4 2B= 092 ‘Top is [3 miles north, 2 miles west, of South Hadley (b) Potting in 2 = -2, f= 10(-12 ~ 12-36 + 36 +84 + 12) (@) Putting in 2 = 1, y = 1: Vh= 10[(2~6~ 18) + (2-8 + 28) 9] = 10(-22% + 224) = 220(-% +9). [Val = 22073 = [311 f/mile} direction: [northwest] 4 CHAPTER 1. VECTOR ANALYSIS Problem 1.38 a= (2-284 -V)FHE- () Ver (b) VQ) @~2P FU vF ey. Rle-2'P + y-vP + e- eed GIF EO (a — a2!) + Ay —y')F+2%z-2')B= 2a, Sle-2) +y-v)P + (e-zyyrtxs Zo-by+ Borde 40-¥afe - 2") - 1-F yy) 9 - 07H - V8 = OMe 2) 8+ Y—Y)FeE- =a] (©) Ber) =n = nh d2e,) = Probiem Lid ++y cos += sing; multiply by sing: Jsing = +y sin écosd + sin®¢. =y sing +2 cos; multiply by cos: Fe0sd = ~y sin gcos $+ z cos". Add: Gsing + Zeos 9 = z(sin? $ + cos? g) = 2. Likewise, Fos — Fsin = y. So Boot 2 —sin gs §} = sind; 35 = cos. Therefore wa,-% + HG = +c0s6(V iv + sin OV A)« \ 5. yr trangforms as a ve we Pe he HEE gona somes, { 90.0 tansin ctor. qed Problem 1.16 ()V-va = gel a) + B(Gas") + P(-2es) 2240-22 =0. WV = Lev) + Seve) + RGze) = y+ 22432, (OV-ve = lv?) + B(Cav + 2) + Lvs) = 0+ (22) + (Cv) = Ae +0). Problem 1.16 Vv Pat R SHEE Py? ea Neg [vle? ty? +2) H ag (ale? tv? + 2)-A] + 2t-8/2)0- Hae 4 OF 4 -3/2)0- day" # + 2(-3/2)()H2e = Br“8(a7 +g? + 22) = 3-8 Br ‘This conclusion is surprising, because, from the cae this vector field is obviously diverging away from the origin. How, then, can V-v = 0? The answer is that V-v = D everywhere except at the origin, but at the origin our esleulation is no good, since r = 0, and the expression for v blows up. Ia fact, V-v is infinite at that one point, and zero elsewhere, as we shall seein Sect, 1.5, Problem 117 B= cosdey Hsing vs; Be = ~sindyy +cos$us. = Be cosh + By sind = (98 + FEB) cond + (HSE + BF) sind. Use result in Prob. 1.14 a coag-+ 3 sind) cos + ($4 0086+ 92 sind) sing — 53 sind + Opp cond = ~ (Se 8 + S$) sind + (S598 + GABE) cass ~ (- 3 sing + 3 cos) ae Be sing + B+ cos) 2 Bie + Be = Se cos? 6+ Gp sin cosh + By sin gcosd + BE sin? + Se sin! cos. So 6 $F sin peosg — 8 sin cos p+ 9 cos? 6 = Ge (cos? 6+ sin? ¢) + Bi (sin? g + 00s? g) = Problem 1.18 zy 2 Oy von=| & & & | =H (0~ Gee) + 9(0+22) + 0182? - 0) = [Hore 4 Dep 4 Hh 2? 3a27 222 gy 2 eo vx0-| & & & |=x0-2w) +5 0-32) +200 - x) = [yk 329-2, ity Que Sze RF @ ©@vx.=|£ & & | = R22 -22) +90 - 0) + 2(2y- 2) = [0] y? (2ey+27) Qyz Problem 1.10 v=y8+429; or v= yek+ 22 +298; oF v= (B2%2— 28) R439 + (2? — 322") B; oF v = (sin2)(coshy) &— (cos.2)(sinh y) $3 ete. Probient 1.20 ©) Vg) = Ya dy + Yoram (p54 off) a (198 +08) 9+ (188+ 996) 2 =S(BR+ HS + He) +0(Sx+Hs+ He) = KV) +10). aed &(AyBs ~ A,By) + & (AeBe ~ AgBz) + $ (AeBy ~ AyBs) 1 OB + BSA — A, Be — BO + A, OBe + Be Ope ~ Ay WB — Behe +An Sp + By Ge ~ Ay he — Be Ge = Ba (hp ~ St) + By (he ~ he) +B, (Fe — He) ~ As (Be Ay (8B ~ 9B) ~ A, (Be ~ OB) = B(VXA)—A(VXB). ged (0) Vx (FA) = (240) — Mh) + (Reed — Sipe) 9 + (Bigs — 2660) g = (s2ft + Ath — 1582 — A,B) + (74 + AGE $88 - ASL) 9 + (s9 + Ay Fe — AL) = 5 (4 - Bt) e+ (He - 9+ (Be - )a ~ [(Avik ~ A08h) 8 (4096 ~ AnBh) 9 + (4085 - 4n 88) 8] =L(VXA)SAX (VS). aed Problem 137 () (AV) B= (4598 + ApS + A.B) 8-4 (Age + Aye + AGP) 9 + (eSBs + Ay Be + As OB) 8 = sfiuf824, Let's just do the z component. #8 +0848) Ties 6 CHAPTER 1. VECTOR ANAL = 3 fe [eet ght tue [bay] +2 [> =hE-A (e+ ay? +22)}= Same goes for the other components. Henet (©) (ver¥) vo = (22 +302 — tae f.) (ayk + ByeH +3222) =a? (yR +09 + 328) + 3x2? (2¥4 22H +04) — 2x2 (OR 4 WH +302) = (vty + 8072?) + (622% 82) 84 Bz (G2? — 2y) 9 — BaPen Problem 122 (i) [V(AB)], = B(AeBe + A,By + A.B.) = eee + AED, + Aye 4 BB, + ALB lane Bin EXE 4B Ay (2B — 98a) ~ Ay (Be ~ Be) [Bx(VxA)], = By(28e ~ SA) ~ 2a) fhe Ath = Ay Ge Ay fe ~ eB + Ate + BO = Byte — DO + Be Ae bBe + y We ABB + Bef fe + Bike + Bip = pe ~ Se + 9h ) + A, (Ge - yp of) B,(— Ble +2 4 Ae) AO + Be 4 Oe) = vas B)], (same for y and z) (i) [Vx(AxB)], = 2(AXB), ~ 2(AxB), = £(AsB, ~ AyBz) ~ §(AzBz ~ AeBz) = Bee, Atk Se, — Aye Boy Ase 4 Bs + Ae [(B-V)A = (A-V)B + A(V-B) - B(V-A)]e Fhe + Bip s Bus Aalhe— Aalbe — Aethe s AalBe + 1B 4 An (— Ope 4 Of 4 Ba + OPs) + By (Mpa — Spe Sty — Ba) + Ay(~B5) + A.(-82) yet) =[Vx(AxB)), (same for y and 2) Problem 1.23 Visa) = RUN e+ Hsia + Hsia) sit s Sf ny HIE 9 4 HIE g fo (Yas BF + Be) — (Gee B+ Bes)] = A get VAla) = de(Aa/9)+ By(Ay/9) + Be(Aa/a) Sea Aedt oR CAGRE 4 Stated = epee OTR oli Adt = plo (He + ee Me) ~ (els + Aye +A QE)] = AAG, ged +8) ~ Be e+ Bhs + Me) 8 (Vx(A/gl, = Rao) =/0) — EG/o = Maa = alee 2) (1-4) = MPKAL SANTIS (same for y and z). ged Problem 124 a zy Bs by ~22 0 V(AXB) = (@) AXB= = 262) + 9(92y) + 2(-22" ~ 6y?) £ (6x2) + Z(G2y) + $(—2e? — Gy?) = 62 +92 40= 152 Vxa =% (£62) - £@y)) +9 (A) - £062) +2(£@)- £@) = VxB = & ($(0) - B(-22)) +9 ($(3u) - BO) +2 (¥(-22) ~ HGv)) = V-(AxB) 4 B-(VxA) — A(V xB) =0— (a= = (b) A-B = 32y — 4zy = —zy ; V(A-B) = BVxA) = 54; A(VXB) = -152 = V(-2y) = 8 (20) + 98-20) =v R29 ee Ax(VxB)=| 2 2y 32 (—10y) +.9(52); Bx(VxA) = ons (A-V)B = (2f +208 +328) (Gy 209) = X(6y) + 9(-22) B-v)A= (vk - 2a) (@R+ 2G + 328) = R(By) + 9(—42) Ax(VXxB) + Bx (Vx A) +(A-V)B + (BVA, = lyk +509 + 6yX~ 2x h + ByK— 42H = -yR—2¥ = V-(A-B). (©) Vx(AxB) = 8 ($(-22" ~ Gy") - $029) +9 (F622) = (-12y ~ 9) + 9(G2 + 42) + 4(0) = “21yR + 1029 He) + $(2u) + BGs) = 1424 (BV)A- i we paces =x Preble 5% = (e) Sh =2; FB = $% =0 = [VT =2. ws aa © &% ® ~ £(-22? - Gy) +8 (1) - $(622)) VA i) VB = (Gu) + H(-22) = B(V-A) = Sy X~ 429 — GyR + 2eF ~ 18yR+ 22H = -yR+ Ory = 9) = 9% --7, = [Ph zsinysine = 261. Be = -167, 1 3 = - 8 CHAPTER I, VECTOR ANALYSIS Problem 1.26 Problem 1.37 2a oe 2) oxo =| g | Be by Be = 0, by equality of croae-derivatives. In Prob. 1,11(b), Vf = 22y224&-+ 3222249 + 42°42? 2, 50 z y 2 VX(VA=| ow 2ayszt Sztytzt 4zty*28 2 8(3- du y?28 — 4 BatyPe?) + 94 Deyte® — 2- day's?) + 6(2- Bays! — 3. 2ey?2t) = 0.0 Problem 1.28 (2) (0,0,0) > (1,0,0). 2:0-+ Ly == (1,0,0) — (1,1,0). 2 = 1,y:0-9 1,2 =05 (1,0) 9 (1,0). 2104 kdl = dzaiv Total: f-v-dl = (1/3) +0+1=[4/3. (b) (0,0,0) + (0,0,1). 02:01; (0,0,1) > (0,1,1).2=0,y:041,2= 1,1) G1). 220-41, Total: [vd = 041+ (1/3) (e) 2S y = 2:03 Ide = dy = dz;v-dl Sv-dl= ff 4x? dx = (40°/3))Q, (@) fv-al= (4/3) ~ (4/3) = (0) Problem 1.29 ay :0 + Lz = Oda = dedya;v-da = y(z? - 3)dedy = ~3ydedy;fv-da = ~3f2dz f2ydy = ~3(a8)(G18) = —32)@)_ = [12] In Ex. 1.7 we got 20, for the same boundary line (the square in the 2y- plane), so the answer is [n6:] the surface integral does not depend only on the boundary line. The total Aux for the cube is 20 + 12 Probiem 1.30 JT dr = J 24dedydz. You can do the integrals in any order—here itis simplest to save = for last: ff (e)a« | sothe z integral is {(0-*-" de 1 2)y- F/B? | = (Bb - Bh) +908 deX,v-dl= 22de; fv-dl= [2 2%de = («°/3))} = 1/3. = dy§iv dl = tyzdy = 0;fV-dl =0. paz = de, fved= fide =a 1 fv edd = 0. 2ydy; [v-dl = fy 2udy =v" = fiztdz = (x*/3))h = 1/3. ppv dh = ytd dl = dy 95v-dl = 2y2 dy Rvedl= 2? de; fv. dl a2 de + 2yz dy + y? dz = 22 de + 20 de +23 de = 42" de -y~2, For a given z, y ranges from 0 to (i-2? -(- 27/2) =~ 2)7/5 ‘The sloping surface is 2+y-+2 = 1-2, 80 the y integral is ff""(1—y~ 2) dy aya)- pa Finally, the z integral is ff 2°(}— 24 )de= M(B - 8+ 8) ar = (8-44 Hb = -+4 /60. Spite 1.31 T(b) =14+44+2=7; Tle) =0. > [T(b)—Ta)=7. = (On + 4y)R+ (de + 22°)9 + (6y2")85 VTodl = (26 + Ay)de + (A + 22 )dy + (6y2")de (@) Segment 1: 2: 0-41, y= 2= dy = de =0.[VT-al= fo(22)de = 2*[) = 1. Segment 2: y:0-1, 1. SVT adh = fi (4) dy 4p PVT =7.0 Segment 3: 2:01, . UT -dl = fo (622) dz = 225), (b) Segment 1: 2:01, eta Lids =0. Segment 2: y:0-+1, 2=0, 2 OSVTA = [(2)dy = 9h =2 | wong Segment 3: 2:01, y=z=1, dy=de=0.fUT«dl = fii2z+4) ae LTA a7 ¢ = (et +42), ; (2:01, yaa, 222%, dy =dede = dds. VIdl = (22 + dz)de + (Az + 22°)de + (G22")2zde = (102 + 142°)dx. SE VT dl = ff (102 + 142*)de = (52? + 227)|) Problem 1.32 Vv sy +224 32 u SiVevdr = fly +22 +32) de dy de = Jf {elu + 2e + 32) de} dy de > [ut 22)2 + $27]9 = 2y +22) +6 I{Ietu+ 42+ 6)dy} ae SS [+ e+ Ou] = 4 +2(42 +6) = 82 + 16 0 AR(Ge + 16)de = (42? + 162) [2 = 16+ 32 = [FE] Numbering the surfaces as in Fig. 1.29: dy dz. [veda = ffydyde = 2422 = 8. (i) da = dyde3,2 (ii) da = ~dy dz, 2 (iii) da Gv) da (v) da dzdrds. fueda = [fd4zdedz = 16. da = 0, veda = 0. veda = 62 de dy. fveda = 24. —dedya,z = (vi) da = veda = 0. fveda =0. = [veda =8+ 16424 = 48 ¥ Problem 1.33 Vxv = 8(0—2y) +9(0— 32) + 2(0—2) = -2y%- 329-28. dz, if we agree that the path integral shall.run counterclockwise, So (Vxv)-da = ~2y dy de. 10 CHAPTER 1. VECTOR ANALYSIS Serxvjda = f{o-*(-2u)dy} de vp? = -(2-2) -Ra- fet 2)ds =~ (42 ~ 242) -(8-8+)=[4] By Corollary 1, 1 Oda should equal $. Vxv = (42? ~ 22) +223. 5 20 1, (Wxv)eda = (422 — 2)dyde; JV xv)eda = f(42? - 2)d2 4 ny 091. (Vxv)-da=0; f(Vxv)-da = 0. 1; 2,210 1. (Vxv)w 1s {CV xv)-da = 0. 82:04 1 (Wxvida =O; [(Vxv)-d 5 2,y 2091. (Vxv)-da = 2dedy; f(V xv)-da = 2. 2 [iV xvpda =} +2= 4.0 Problem 1.35 (s) Use the product rule Vx(fA) = (VA) ~ Ax (Wf): [t0rxay-aa= fvxcray da firs cwp)-dam f saas fin xcopy) da. ed. (used Stokes’ theorem in the last step.) (b) Use the product rule V-(A x B) = B-(VxA)—-A-(VxB): [B-Coxaydr= [ornare [aC xB)ar (L used the divergence theorem in the last step.) (Ax B)-dat [A (VxB)dr. ged. un Problem 1.36|/r= 24747; 0 Problem 1.37 - (==): @= tan" (2) ‘There are many ways to do this one—probably the most illuminating way is to work it out by trigonometry from Fig. 1.36. The most systematic approach is to study the expression: F=sk+y9 +20 =rsindcosp+rsindsin 9 +r cos0a. ET only vty slightly, then dr = 2(e)dr is a shor vetor pointing in the direction of increase in r. "To make it a unit vector, I must divide by its length. Thus: BE = sindcosdk + sinOsin dy + cosa; | 2} 95 = rcosd cos pk -+ r cosOsin gy — sind a; $ = -rsinsing + rsindcos99; || = sin? Ocos? g + sin? @ sin? $ + cos? @ = 1. 1 cos? 8 cos? $ +1? cos? Osin® g +r? sin? 6 = 7? gin? @sin® $ +r? sin? @cos? ¢ = r?sin? 4, F = sind cos ph + sinOsin 69 + cos03. 16 = 038 cos % + cosdsing 9 —sin#z. G =~ sings + cosy. Check: FF = sin? 6(cos? $ + sin® 4) + cos? @ = sin? @ + cos@ = 1, ¥ 6.6 = —cosdsing cos + cosBsingcosd =0, ¥ ete. sin f = sin? 9 cos + sin? sin 49 + sin cos 02. 0546 = cos? 8.05 6% + cos? Asin} 7 ~ sin8.cos0 2. Add these: (1) sind# +0050 = +cose2+ sings: @) in dR + cosd9 Multiply (1) by cos¢, (2) by sin g, and subtract: Multiply (1) by sing, (2) by cos, $= sind sin g# + cos@sin ¢6 + cosdd. cos8 = sin 8 cos 6 cos $% + sind cos6 sin PF + cos? 4a. sin 96 = sin 6 cos® cos + sind cos sin dy — sin? 82. Subtract these: —sing6. = 8 we CHAPTER 1. VECTOR ANALYSIS Problem 1.38 @) Ve = A Pr) S0Vovs)dr = f(4r)(0? sin 8 cdr ad dd) = (8) Jj Pdr Sy sin 848 f3rdH = (4) (HE) (2y(2m) =[ae RE Svreda = f(r?t)-(r? sin 8 a0 dg) = r4 J sin a8 [2 dp = 4 R4 / (Note: at surface of sphere r = R.) (0) Vove = bg (2) =0 = []lVevnar =0 Lveeda = [(4f) (r*sin8 d0 8), 0 (if a 36 = 62, s0.€ is outside Y, so the integral is [Z=rO1] (@) (e- @+29 +28)? = (18409 + (-1)8)? =141=2< (15) and hence the integral is e-(d — e) = (3,2, 1)+(-2,0,2) = -6-+0+2= Problem 1.48 First method: use Eq. 1.99 to write J= fe~* (4n6%x)) dr = 4ne~ Second method: integrating by parts (use Eq. 1.59). 25, 60 ¢ is inside V, w 4 -[# vernars fe Fda, But V(e~" = f Reva ars fer$ rainoandse tn ferrarser® [smoavas cates =o) = dn (ery tre ® = i (ne pe Problem 1.49 (a) V-F; = +20 +2 (@) =) VFs= e+ H+ Ha1s1t1 o : UxP2= 8 ew = ape eRe ogee VxFi | — [Fa is a grad i Fyisacurl] [t=3 (2? +y?+22)] would do (Fa = U2) For Ai, we want (2 ~ S44) = (Sf — %) <0, Sh Ot oat 2) A, = A, = 0 would do it. [i= SF] (Ps = VAD). ut tee ae nat unique) ey 2] (b) V-Fs = £(u2)+ Zlex) + Ploy) &E - a) +9 (yy) +2(2-2)=0 ve ote oy So Fa can be written as the gradient of a scalar (Fs = VU) and as the curl of a vector (F's = VxAa). In tact [Us = sz does the jb. For the vector potential, we have fu — 84x < ys, which suggests A, = ty2z + fle,2); Ay = —dye? + (9) Soe Ge 2s, suggesting Ay = f2%x + le,y)s Av = ~$22" + J(u?) fe - Se = ay, 50 Ay = het + Aye); A, = —hay? + ltevy) (again, not unique). Protlem 180 7 (@) + (a): UXP = Vx(-VU) =0 (Eq. 144 ~ curl of gradient is always zero). (a) = (o): $F dl = f(VxF) “da = 0 (Eq. 1.57-Stokes’ theorem) (3 Q): [OF edt - JP Fed = [2,F dl + fe, ,F-dl= $F -@=0, 50 [ras [ ra (0) = (€}: same as (c) => (b), only in reverse; (c) = (a): same as (a)=+ (6). Problem 1.51 7 (d) > (a): VF = V-(VXW) =0 (Fa 1.46—divergence of curl is always zero) (a) + (e): $F da = [(V-F) dr = 0 (Bq. 1.56—divergence theorem). (c) 3 (b): J, F-da~ fj, P-da = $F -da = 0, 0 es (Note: sign change because for # F da, da is outward, whereas for surface II it is inward.) (0) = (e}: same as (c) = (b), in reverse; (c)-> (a): same as (a) (c} Problem 1.52 ~ In Prob. 1.15 we found that, V-v, 2 in be written aa the gradient of a scalar; vq can be written as 0; in Prob. 1.18 we found that Vxve = 0. $ ¢ curl of a vector | (a) To find ¢ Q) HP stave 1,2) (2) § = (xy +24) (3) f= uz 16 CHAPTER 1, VECTOR ANALYSIS os From (1) & (3) we get $f = 2yz => f = v2? + ol) + t= y2e + y2? + aly), 50 $F = Qzy +2? + = 2ey-+ 2? (from (2)) => 92 = 0. We may as well pick g = 0; then (&) Tofind Ws SH — Pick W = 0; then Oe = 52%, Ma. OM — ate = gata; Bt Ot FE = whet We = 382 + Hla) 8H, aesee Wye et ain) omy, — 20 2 0. May as well pick f = w Cheek 2 |-2¢ ) + ¥ (Baz*) + 8( 2x2) tz fet. | You can add any gradient (Vt) to W without changing its curl, so this answer is far from unique. Some other solutions: W = 28% 2729; W = (Qeyz 4 x29) & 4 27y a; W = nyzh— Sate 9 + 24 (y— 32%) & Probeim 1.53 1 Fund ag (7? eosdsing) ww = 42 (10080) + 2a © (sind r* cose) = et sind 38 1 _ 2 ai c050 + —=—5 cos? cs 6+ == (—1?cos0 cos) = TE acing + om ~ cos] = drome 2 fp p [ovr = [arcoseyr?sinadr diag = i feetena ae (2) (2) GE = 9G) @)-2E) Surface consists of four parts: (1) Curved: da = Rsinodd dpi; r= R. v-da= (RP cos0) (i? sind d0 46) [v-co=nt fcstanse fone Q@-*. ) y 1 (2) Lefts da = —rdr dO §; 3) Back: da=rdrd0d; v-da= (r¥cos@sing) (rdrd8) = 0. fv-da=0. 2. v-da = (—r?cos@ sin 9) (r dr d8) = —r* cos 8 dr 8. a =p Iv aan [rte | tan—- (Ger) can dns (8) Bottom: da = rsin dr dba; 6 = 7/2. v-da= (r¥cos¢) (rdrdé) Total: §v-da=7R/440- LR + ER = (0). So J(Wxv)-da = (b~ alah, = (ay +b29)- (de +dy9 + de8) = ayde + be dy, 2? +42 = R= dads + 2ydy =O, s0 dy = —(x/y) dz. So v-dl = aydz + ba(~z/y) dz = } (ay? - ba?) dz. For the “upper” semicircle, y = VT =a, so v- dl = Se ae. [va = [EGR ee fontsin (2) (ery [SVR e+ Bann (Z a 1 nt(a— ayant) RY gRi(a~ d)sin“*(2/R) 1 1-1) —sin“ 2 GP @—B) (sin"(-1) ~ sin (41) = dR (0-0) ( lin = FrR-0). ‘And the same for the lower semicircle (y changes sign, but the limits on the integral are reversed) so fv-d=1Rb—a). ¥ Problem 1.55 ()2=2=0; de= =O; y:0-41. vedl=(y+3z) dy = yay, fras fra} 3 a dy; ys 1-40. v-dh= (y+ 8x) dy + Bde = dy ~ 12dy = (y ~ 12) dy or (t-2) =o (Q)2=0;2 @r= 18 CHAPTER 1. VECTOR ANALYSIS Total: fv-dl=4—4412-12 Meanwhile, Stokes" thereom says fv-dl= f(xy) da. Here da = dy dz, so all we need is (Vxve= BO) — Ply +32) =O. Theretore f(V Xv) da = 0.0 Probiem 1.56 ; - | Start at the origin, | () =F, 6=0; r:0-91. vedl=(reo#6) (dr) =0. fv-al=0, (2) r21, 0=F; $:0-94/2 ved =(r)(rsinddg) = 349. fv-di=3 f dp = %. 1,801 = hy, dr = gat condo, 0:5 4% (3) d= Hi rsind =y veal = (rete) ~(rensnair a) = 228 (S288) gata ~ 208 wy sind Sin sin? cos cos 086 (cos? 8 + sin? 8 cos (eee eee ag eee (cee nae Ege owes (Sa See) #= Se (Mae) #= Ss Pherefore ), cos alge eet 1 Gee feo fear [B68 meal, ra Tw a ri VE-00, v-dl= (reos*6) (dr) = dr dr. Total: Stokes’ theorem says this should equal {(V xv) da 1 Fsind a [Scratmn-f rare é Vxv = [Fptsinear) ~ Fe c-rsingcoss)] #+ = pL plbreosale + 2-6r}6+ 4-2rcosdsind + 2rc0s8 sing] = 3cot8#~66. (1) Back face: da = —rdr dd g; (V xv) -da = in dr db, (Vw) da [vw in= fort fare 2%. « JV xy) -da = 0. sin @dr dp. 8 = ¥, 50 (V xv) -da = 6rdrdp (2) Bottom: da = 19 Problem 1.57 wedl= ydz. (1) Left sides = a2; dz=—dr; y=0. Therefore fv -dl = 0. (2) Bottom: dz 0. ‘Therefore f vd (3) Back: 220~ $y; dz =-1/2dy; y:20-40. fv-dl= fiy(-Hdy) = la Meanwhile, Vv = so {(V Xv) -da is the projection of this surface on the ty plane = }-a-2a=02, ¥ Problem 1.58 = d2 es 2 note? cosd) + — eg ® (rt Ver = FF letrsind) + 59g (6nd a? 008) + 5 (tan) Lar sin tr? (cos? 9 — sin®) = *°. (ain + cos? 8 — sin? = pat sind + gt (cos! ie eatceraeeeemt ate) as foomae = | (2) ranoerou = fader Ponto as (09 ff = ant (3,89) = (40) «fer raa] Surface consists of two parts: (1) ‘The ice cream: r= R; $:0 + 2x; 8:0 1/6; da = R?sin@ dé dét; vida = (R*sin#) (R? sin@d9 dé) = Ro sin? @ dO dé. sing) = (- (2) The cone: 8 = %, $0 ~» 2x; r:0-¥ R; da =rsin@dpdr6 = Srdrdgb; v-da = V3r° drdp _ arsfea fons m8 ay [vaaa nt [ snroai [ 46= (A) (25) cE ~ isin Therefore f'v-da = E(g- Gavia af ertav9. Problem 1.59 . (a) Corollary 2 says f(VT)-dl = 0. Stokes’ theorem says $(VT)-dl = [[Wx(VT)]}-da. So [[Vx(VT)}-da = and since this is true for any surface, the integrand must vanish: VX(VT) = 0, confirming Eq. 1.44. 20 CHAPTER 1. VECTOR ANALYSIS (b) Corollary 2 says §(V xv)-da = 0. Divergence theorem says §(V xv)-da = f V-(Vxv) dr. Sof V-(Vxv) dr = 0, and since this is true for any volume, the integrand must vanish: V(V Xv) = 0, confirming Eq. 1.46. Problem 1.60 a ~ (a) Divergence theorem: fv -da = f(V-v)dr. Let v = cP’, where ¢ is a constant vector. Using product, rule #5 in front cover: V-v = V-(eT) = 1(V-e) +e-(VT). But e is constant so V-e = 0. Therefore we have: Je-(WP) dr = fTe~ da. Since e is constant, take it outside the integrals: ¢- f VI dr = e-[T da. But is any constant vector—in particular, it could be be %, or ¥, or —S0 each component of the integral on left equals corresponding component on the right, and hence aed (b) Let v > (v xe) in divergence theorem. Then f V-(v x e)dr = fv x ¢)- da, Product rule #6 > V.(v x @) = (Vv) —v-(Vxe) =e: (VXY). (Note: Vxe = 0, since e is constant.) Meanwhile vector Identity (2) says da (v x e) = 6 (da x v) = ~e- (v x da). Thus fe-(Vxv) dr =~ fe-(v xa). Take © outside, and again let be &, §, # then: forsee [ord ood (0) Let v = TVU in divergence theorem: f V-('WU) dr TV(VU) +(VU)- (VT) = TVU + (VU) (VT). Therefore [TVU-da. Product rule #(5) = V(TVU) = fave +(VU)-(VT)) ar= fervuy da. qed (a) Rewrite (¢) with T 4 U: f (UVT + (VT) -(WU)) dr= SUV: that the (VU). (VT) terms cance: da, Subtract this from (¢), noting / (rv?u -Uv?r) dr = f (VU -UVT)-da. ged (©) Stoke's theorem: [(V xv) da = fv-dl. Let v = ef. By Product Rule #(7): Vx(eT) = T(V xe) ex (VT) =e x (VT) (since © is constant). Therefore, ~ [(e x (WT)) -da = f Te~dl. Use vector indentity #1 to rewrite the first term (c x (VT) -da = e-(WI' x da). So ~ fe-(WT x da) = $e-Tdh. Pull e outside, and let ¢ ~> &, §, and @ to prove [ornd=- fra oot Problem Li (a) da = R? sin dd dp. Let the surface be the northern hemisphere. ‘The and ¥ components clearly integrate to 2er0, and the & component of Fis cos8, s0 x2 aents [ {b) Let T = 1 in Prob. 1.60(a). Then VT =0, 50 fda=0. qed. (c) This follows from (b). For suppose a; # a2; then if you put them together to make a closed surface, fda =a; ~ a £0. (@) For one such triangle, da = }(r x al) (since r x dLis the area of the parallelogram, and the direction is perpendicular to the surface), so for the entire conical surface, a =} fr x dl. ax [ Rsindcosodoaye a1 i (©) Let T = c-r, and use product rule #4: VT = V(e+r) = © x (Vxx) + (e+ V)r. But Vxr = 0, and VE = (ek bey Her Bek + US = 28) = re + 6,9 +e, 8 =e. So Prob. 1.60() says fra = fee naa form nta=-fords=-cx fin=-exazexe oad Problem 1.62 a) 1 ‘=a ) Por a sphere of radius Ri fv-da = J (4f)-(RPsinod9 dbz) = R fsinododd = 4nR. | FE So divergence [(Vvjdr = () (r*sinodraods) ({+) (fsinododg) = amr. ‘theorem checks. Evidently there is no delta function at the origin. 10 pam 18 on "= Fa, (except for n = 2, for which we already know (Eq, 1.99) that the divergence is 48%). Aloe aye fara Vx ("= (?) (2) Geometrically, it should be zero. Likewise, the curl in the spherical coordinates obviously gives To be certain there is no lurking delta function here, we integrate over a sphere of radius R, Prob. 1.60(b): If Vx(r"#) = 0, then [(Vxv)dr = 0 2 -$v x da. But v = r"F and da = — A? sin 8 dé d@# are both in the # directions, so v x da = 0, / 5 Chapter 2 Electrostatics Problem 2.1 (a) [Zero, 1 | rer Explanation: by superposition, this is equivalent to (a), with an extra —g at 6 o'clack—since the force of all twelve is zero, the net force is that of ~g only. (©) [Zero.) (€)| 2-29 | pointing toward the missing g. Same reason as (b). Note, however, that if you explained (b) as ofr where ris the distance from center to each numeral, F points towsnd the mising ¢ | fre © ancellation in pairs of opposite charges (1 o'clock against 7 o'clock; 2 against 8, etc.), with one unpaired q then you'll need a different explanation for (d). cs (a) “Horizontal” components cancel, Net vertical field is: B = giz8 cosé. Qgz iS 2 EHO | : When z > d you're so far away it just looks like a single charge 2g; the field agleg should reduce to B= 2-48. And it does (just set d 0 in the formula). (}) his time the “vertical” components cancel, leaving = sighs sind, or 1 ad ee Or (Dy A. % %, which, as we shall see, is the field of a-dipole. (If we From far away, (z > d), the field goes like B= 7% set d+ 0, we get E = 0, as is appropriate; to the extent that this configuration looks lke a single point charge from far away, the net charge is zer0, 60 E+ 0.) 2 23 Problem 2.3 Be = gy [Ef con6; (0? = 2? + 2%; cond =f Satins . wal lA pet Al(a+ For 2 L you expett it to look like @ point charge q™= AL: B+ zi; %48. It checks, for with 2 >> L the & term -» 0, and the Z term -» 71-24 Problem 2.4 - ~ 7 From Ex. 2.1, with L + $ and z+ y/z? + (8)° (distance from center of edge to P), field of one edge is: BR a reo fate eft ea ‘There are 4 sides, and we want vertical components only, so multiply by 4 cos@ = 4—t— ae ‘ahaz - Pa) e+ Problem 2.5 “Horizontal” components cancel, leaving: B= z- { [4 cos6} & Here, 2? =r? + 2, cos = £ (both constants), while fal = 2nr. So a a (aan __ Abas | BS ee [Po eae Problem 2.6 : 7 ~ Break it into rings of radius r, and thickness dr, and use Prob. 2.5 to express the fielé of each ring. ‘Total charge of a ring is o- 2nr dr = + 2nr, so A= odr is the “line charge” of each ring. 1 (odr)2nrz 1 # Fine = Fre ty k= Bega | faa 1 araz |} ~~ 2 Ve 4 CHAPTER 2. ELECTROSTATICS For R > z the second term + 0, 80 Epiane = 75208 Fors > Ry ge = 2 (1+ BY wt and B= Gh Re = 3, where Q Problem 2.7 Ro. Bis clearly in the = direction. From the diagram, oda = oR sin9d0 dp, R +2? —2Rzcosd, 1 [oR sind db dglz ~ Reosd) Greg | (P+ OR cos” = (orrtay [” (= Rens) sind + Fg rh of (RE +2? — Rez cos)? 7 z-Ru jy (R? $2? = 2Rzu)s? = errtay fb R__)’ __1_ 2xRo ((z—R)_ (-z- A) ana sz). Treg 2? {€ RY PR} For : > R (outside the sphere), E, = shri = gh &, so Gh 4 For z < R (inside), E; = 0, so (B=0.] Problem 2.8 According to Prob. 2.7, all shells interior to the point ({.e. at smaller r) contribute as though their charge were concentrated at the center, while all exterior shells contribute nothing. Therefore: Jap = on 8. = geet) au ner can dn y pt ator kok up 1 Qine EO= aa where Qigs is the total charge interior to the point. Outside the sphere, all the charge is interior, 90 1a, B= 70 S+ Inside the sphere, only that fraction of the total which is interior to the point counts: Led 1@ neg Qint Problem 2.9 (0) p= @ V-E= aed (r? -~ (b) By Gauss’s law: Qeae = 0 § E-da _By direct integration: Qune = dh Problem 2.10 ‘Think of this cube as one of 8 surrounding the charge. Each of the 24 squares which make up the surface of this larger cube gets the same flux as every other one, so: 1 [oaueh [ve ‘o(RR®)(4eR?) = [4ncok R®.] Af Seghr®) (Arde) = 20rc0k f hdr = dnegk Ro. ‘The latter is L9, by Gauss’s law. Therefore. LE da = 5h Problem 2.11 Probiem 2.12 r~ ‘Gaussian surface Problem 2.13 = eee = = Gaussian surface ae $B-da=E-2ns-1=1LQme= LN. So = a ipo —— B= 548 | (ame as Bx. 2.1) r [P= Grees' ae Problem 2.14 —_ Gaussian surface $d = 26 CHAPTER 2. ELECTROSTATICS Problem 2.15 () Qene = 0, s0 [B= 0.) Ij? sin aradd phi iE) Problem 2.16 pst; 0 CQ) Gaussian surface Gaussian surface Oo) $B-da=B2ns-1= 3Qene = Lpra’l; (i) fE-da=E-2r8-1= 2 (B=0] a aT On the zz plane B = 0 by symmetry. Set up a Gaussian “pillbox” with one face in this plane and the other at y. Pi Gaussian pillbox f E+ da ' 27 Problem 2.18 From Prob, 2.12, the field inside the positive sphere is By = r+, where ry is the vector from the positive center to the point in question. Likewise, the ld of the negative sphere is ~ 32. So the total field is 2 Beg ht FD But (see diagram) ry — Problem 2.19 ava dtr hf fox($)]odr ance pdm on #02) ed Arco =0. (since Vx (3) =o, fom Prob. 1.62). Problem 2.20 zy 2 QQ YxEL =hlés gy Be | = RRO ~2v) + 90-32) +200 - 2)) £0, zy ye 322] so E; is an impossible electrostatic field. @) VxE, K[&(2z ~ 22) +90 - 0) + 82y ~ 25) 50 Ey is a possible electrostatic field. : Let's go by the indicated path: Bs d = (y? de + (22y + 2")dy + 2yzds)k (20, 0 20) yt de = 0. Fi 28 CHAPTER 2, ELECTROSTATICS E+ dl = 2hyz dz = 2kyo2 dz. Sur B+ dl = 2yok f° 2 dz = kyoxs. esap zo) V(z0,Y0) 20) = ae el = —K(zoug + ¥oz$), or |V (zy, Check: ove feles4ys?) 84 & lav" tycth 94 de (eateve) Problem 2.21 ~~ Vo) =- JL Bea. { 2 a4 aaytst)942y28)=8. Outside the sphere (r > R): B= FL Se. Inside the sphere (r < R) oir? Soforr> #: V(r) =~ fz, (ast) = aaa (Hl, =| go> and for < Ri V(r) =~ [2 (aks) oF - Ja (agar) ar = ah When r > R, WV = aig & (LF a VV = go hrty Jaf 50 = zig 28s (Prob. 2.13). In this case we cannot set the reference point at oo, since the charge itself extends to o6."Let’s set it at ¢= a. Then Vos) = =f (sag) a= Fan) (In this form it is clear why a = co would be no good—likewise the other “natural” point, a = 0.) VY = ~ ag ?Ads (in (5)) 8 Problem 2.23 V(0) = ~fQ Bed = ~ [L(A P)ar ~ (ALS ar — PPO)ar = EES =£{1~$-in(g)-149) = Kin() Po Problem 2.24 Using Eq. 2.22 and the fields from Prob, 2.16: V0)-V(0)=-B-al=- RE -a- pte. Seeds — 9% f' ds Problem 2.25, L2 Oe Ee wag les VPFP) +t (OV = a It Bets = gino (VF FRE = In each case, by symmetry $Y = $f = 0. (a) B= ~ ara (~4) (* eta) a 1 | (agrees with Ex. 21) © [Steg VF EE i () B= as {arpybemy ete Gi B= a5 {hype - [Eb seal oo —a If the right-hand charge in (a) ic ~q, then [V = 0], which, naively, suggests E = VV = 0, in contradiction with the answer to Prob. 2.2b. ‘The point is that we only kaww Von the 2 azis, and from this we cannot hope to compute E, = ~9¥ or E, = ~¥. That was OK in part (a), because we knew from symmetry that By = E, = 0. But now E points in the 2 direction, so knowing V of the 2 axis is inslficient to determine E. —/> Problem 2.26 VN odar no Vo) = f (=) em ean = ie (where ¢ = 2/3) vo) = a) dh, wheres = yh? +? Ve ano 1 wa, -mal yo wag [Vinnie Ste is vin sv h + in(an+2v2h—v3n) — h— Ge eme vm Jo i o_hp ] Finan VBn)| = =F ncn vBh) — (2h VI] a =a lt = (54) a ohay ¢ o) oh inc + v3). 2- Va} * Xe 20 30 CHAPTER 2. ELECTROSTATICS Problem 2.27 E L Cut the eylinder into slabs, as shown in the Bigure, and — use result of Prob, 2.25c, with 2 2 and o + pdr: #55 un a) - ved, J (VR 4s -2)dr J ue l= =i} EVES ene + VER) — my =e Woncenon = late (onvincrr c (Wote: — (2 + §)? Problem 2.28 Orient axes so P is on 2 axis, aa Here p is constant, dr = 1? sin 0dr d@ de, = gh seer V= are Sia Cees Vm gli SPenagtetele | Pag = 2 vars SS perch cmeg = (VESTA Der cOO)y = (VEER Drs ~ VFR TR) sherpa (ese : R an off rears Pier) = But p= prss0 V(e)= dhathe (0-4) = sta 0 8):[¥ a1 Problem 2.29 VV = aS VYY(E)dr = abe fole’)(V*E)dr (since p is a function of r’, not r) = ag Sole’) [-40 dr ~ var = ~ ole). Problem 2.30. (a) Ex. 2 Babove = 38; Byetow = — 265 (ft always pointing up); Esbove ~ Evetow = Ex. 2.5: At each surface, = 0 one side and E = t ather side, so AE = £.¥ Prob. 2.11: Egy = 2836 = £F; Big =0;80 AE = 26.4 (b) Outside: fE-da = E(218)l = LQene Inside: Que = 0, 50 B= 0. «SE (6) Vest (at surface); Vin = 8% ; 80 Vou = Vine ¥ Yow = — Bg = —E (at surface}; Oia = 0 ; so You - Ha =~ (at surface). Problem 2.31 @V = ae Lam Wy = qv =| (b) Wy = 0, Ws = £); Wa = (ee (a). Wore gf {14 gp- 1-24 ep = wet (+5): 32 CHAPTER 2. ELECTROSTATICS Problem. 2.32 (a) W =} foVar. From Prob. 2.21 (or Prob. 2.28): V = ¢ lq “(0 $) = ok (3- ) rear ° i Si -&(-F) a (0) W = $ SE%dr, Outside (x > RY B= 5 Inside (7 < R) B= ger amet {If Het {C5} ore} (Mra Ol) - (c) W = #{ f,VE-da+ f, B’dr}, where V is large enough to enclose all the charge, but otherwise arbitrary. Let's use a sphere of radius a > R. Here V = pi; ao la oe 5) sin 5 west f( a!) \(S) avdo~ [ ~$ (aoe att Le (La wil se) nae lata ita} Fro 5 RY As a + 00, the contribution from the surface integral (3 (ae ECG - D) picks up the slack Problem 2.33, £2) goes to 220, while the volume intgel av cay =a5(<2) 4 (ncn hereto Problem 2.34 was Ba B= ghd (a 0). So By Ex = (gig) 3h, (7 > 8) and hence [x -Badr = ~ (hz) ofp? Seter2dr = ~ yi Woo = Wi + Wo +0 JB, Brdr = poe (E+$-9) =e (E-PY Problem 2.35 : (lox () VQ) =~ f,B-d Sool ates B)4r ~ Jo Ode ~ £3 (aaeg te) ar ~ Syd = mG+s a live G+ %- 2) (0 BY te care “enn of); 10) = = 2d — Later — fe = (2-2) ] Problem 2.36 @|o0 ea as Bai [n= SP ate (0) [Bow = | where r= vector from center of large sphere. * ine in fis, | where ra (rp) is the vector from center of cavity « (Q). e (6) an changes (but not ge oF a); Eyussde changes (but not Bs or Es); force on ga and g, still zero. Problem 2.37 Between the plates, E = 0; outside the plates = o/ey = Q/eod. So = Opi 2 |e et cre br Problem 2.38 Inside, E = 0; outside, E = <4 i Exe = bgle BA fe = 0(Bwelsi o = aha. Fee Sfeda = flqSq) (che) cos R sind a ag =i (a ‘on fo!” sin cos do = 3-9)? (4sin?9) |<” = M4 CHAPTER 2. ELECTROSTATICS Problem 2.39 ‘Say the charge on th JE-da=E-2ns-L= 2 er cylinders Q, for a length L. The field is given by Gauss's law: QE = 52718. Potential difference between the cylinders is [v-a=-s&; [ ta=-Sp9(2). [As sot up here, ais at the higher potential, so V = V(a) ~ V(8) = 5,2 In(2). V(b) - V(a) = C= $= BE, so capacitance per unit ength is Problem 2.40 (2) W = (Force) (distance) = (pressure) (area) x distance) (b) W = (energy per unit volume)x (decrease in volume) energy lost is equal to the work done. Problem 2.41 From Prob. 24, the field at height 2 above the center ofa square loop (side a) is 1 Adar haere (are Here A+ of (see figure), and we integrate over a from 0 to a ug Let u =", s0 ada = 2du ow : ede ote ed 1 ay oz : 5 tan a4 get [ Gre Vinee ~ reo ) =e} 4 60 (infinite plane): B= 24 [tan (00) 2 >a (point charge): Let f(z) = tan“! VIF ~ §, and expand as a Taylor sesies: f@)= JO) +2/'O+ 5 2 f"(0) ++ _~Here f(0) = tan(1) - $= F- F =; S'(e vise = apse 9° S'0) = 1 2 3 fet Os? + Os ‘Thus (since gy =2 <1), Be 28 (Ih) = hg Problem 2.42 cavepeg{ ho (24), 1 2 (Bsindeose ramets 7) as rt} 7 Being) Problem 2.43 From Prob. 2.12, the field inside a uniformly charged sphere is: E = ;- rr. So the force per unit. volume is £= pB = (7S) (geSar)t = 2 (g9n)"t, and the force in the = diretion on dri 3 qa) 2 r= 2 (jp) reste sna dean ‘The total force on the “northern” hemisphere is: = [item 2 (Qa) [eu ff” croanoa [a 2 “3G@e) (A)(S : 1 o da = 2nR? sin 6 8, se [devin {t= Pa ; Treo aR +R eee g L sexe "8 sindd _ oR = plas Ve OMT awa” oR Vpote ~ Veenser =] 57-(V2 ~ 1). Problem 2.45 First let's determine the electric field inside and outside the sphere, using Gauss's law: akrt (F< R), 36 CHAPTER 2. ELECTROSTATICS Sob=Brt(r R). Method wa [ear cea.206)= 9 [" (M2) area [° (ME) eee 9 (6) [flomen ae] Fee (DL) (Ee) Method tt wel [over (x22) mrsneoe Loan fie Lord (eC) ear Pal al fool att aoe a Problem 2.46 af Neal =faorarane| V (eG An) }. But V: (4) = 4n6%(r) (Bo. 1.99), and = [pdr= oa {u/Poe = 8 [Pantin = soa (ta atte freer) But [p°re"*dr = sh, 90 Q = Anco (1%) = ao] Problem 2.47 (a) Potential of +A is Vp = — 54 Potential of ~A is here 54 is distance from Ay, (Prob. 2.22). here 8 is distance from 2. 37 (i ‘The equation for a circle, with center at (yo,0) and radius R, is aoe += R, ee a Byidently the equipotentials are circles, with yp = a (£4) and Bop Hayat =o? (f8h)* ot = pattem ann tems ot ee ener Senter [on (ES) ] Rete t wera cea * coun (2228) Problem 2.48 (a) VV = £ (Ba. 2.24), 90| SY (0) QV = bm? + (0) dq = Apde ; 4 = apt T] (constant). (Note: p, hence also I, is negative.) a 38 CHAPTER 2, ELECTROSTATICS () &¥ = -d0= 24 = ~Lay/ae 3 | Ge = OV °2 |, where 8 = ~ a fF « (Note: is nepoive,s0 8 3s postive; qi postive] (e) Mattiply by v= ye movin [vray p [vay BV = 250°" conta bin ¥(0)= V0) = 0 (aod i a potential zero, and el at ahd i zr), ste constant 0 an 2150.8 a fAvis ag vhay = aye fv MA ay = vB fac = 49H = 2/Be + constant. But V(0) = 0, s0 this constant is also zero. (va)"“2, or V(2) = (30) Interms of Vo (instead of 1): | V(x) (3) Geert) ‘Without space-charge, V would increase linearly: V(z) = Vo ( vve 3 7 5 B2, 80 V( 81m)" 43 2A ole = Forjere| (b) [Wes.] The field of a point charge at the origin is radial and symmetric, so Vx = 0, and hence this is also true (by superposition) for any collection of charges. © ee [ea af Eset reread (hemes [2 vo (eee) ail = Vg 14 fear a} 39 ~ £ [Par — exactly right to kill the last term. Therefore vo {= [} - fra (7 @ fb-a me (fe oP gg pe = Now Jare""/¥ar an R Vera Pe gedp = Lf re-eldr ia ares elo. orden (ue 8) aa) (c) Does the result in (4) hold for a nonspherical surface? Suppose we x make a “dent” in the sphere—pushing a patch (arca R?sin @d8 dé) from radius R out to radius 5 (area S? sin 4 dé dg) q afe dam {s (+ Se eI(S? sind adds) — #0 ae f) eee sinoanas)} - = [Ge gle (1+ fa] sneaoas, Snr? sind dr dp cs ie snedodg is rea as inadbae (“ (145))f = [lem (Dem anoaas So the change in 3 {V dr exactly compensates for the change in fE-da, and we get q for the total using the dented sphere, just as we did with the perfect sphere. Any closed surface ean be built up by successive distortions of the sphere, so the result holds for all shapes. By superposition, if there are many charges inside, ‘the total is 2Qune- Charges outside do not contribute (in the argument above we found that © for this volume fB-da+ 3; fV dr = 0—and, again, the sum is not changed by distortions of the surface, as long as q remains outside). So the new “Gauss’s Law” holds for any charge configuration. (0) tn differential form, “Gauss law” reads: | V-E+ 557 = 0, “squashing” the ellipsoid down to an ellipse in the zy plane: 1 o(z, a ea, 1) = Fab Flay = GF (I multiplied by 2 to count both surfaces.) and let r= VFFF [ot pos (a) For the circular disk, set a and then take the limit b -> co: (b) For the ribbon, let Q/ (@) Let b= 6,7 = Vy? FH, making an ellipsoid of revolution: a(z) = ‘The charge on a ring of width da is dq =o2nrds, where ds = Vda? + dh? = de/1+ (dr/dzy. a br dr ir er Age Now 228 , 2dr sods = dey SE ae ATE TFTA, Ths eneerc. ar a _—__ ay @ g i] Mz) = Fh a oer 2 tS ata ‘onstant! (=a eee Fre FRIe = -[g tant!) i oy r / ! : « : oF Na ® . Chapter 3 Special Techniques Problem 3.1 ‘The argument is exactly the same as in Sect. 3.4, except that since 2 < R, V7 FRI=UER i fence Vive = Iie 4-H) = (R-2 Instead of (2 R). Hence Vie = gosto [le +f) ~ (R= 2) R-2), If there is more than one charge inside the sphere, the average potential due to interior charges is and the average due to exterior charges 15 Veemer, 80 Vave = Vownter + it. Problem 3.2 ‘A stable equilibrium is a point of local minimum in the potential energy. Here the potential energy is gV. But we know that Laplace's equation allows no local minima for V. What looks like a minimum, in the figure, must in fact be a saddle poin., and the box “lesks” through the center of each face. Problem 3.3 Laplace's equation in spherical coordinates, for V dependent only on r, reads: oa nt oe =e (constant) = Brample: potential ofa uniformly charged sphere In linda coordinates: V'v = 12 (s57) <0 of Brample: potential of a long wire. _ Problem 3.4 Same a8 proof of second uniqueness theorem, up to the equation J VaEs + da cach surface, either Vj = 0 (if V is specified on the surface), or else Ba, = 0 (if 9% Jy(Es)? =0, and hence Ey = Ey. ged Problem 5.5 Putting U = T= Vp into Green's identity: _ V+ UHV) dr= J WOH da Bu VV = VY — vs y fs 80 ff Bide =~ f Yok, da, andthe tests the same as before, 5 A — fy(Es)? dr. But on ~E, is specified). So 2 =o, and Vi = Bs. 2 /-~Problem 3.6 Place image charges +29 at —dand ~q at z = ~3d, Total force on +4 is, ig 313: 2 = VP Ea a Traces, = [2 = Greg (Gay? * Gaye * Gaye 1 (20), Treo (72a) * = VP+E RFCs. Therefore: Problem 3.7 (a) From - =z (Eg. 318), white b= = (Ba, 3.16). 4 [(ag)? + Re — 2racose ‘Therefore: =-1(9,¢ 1 Von) = aa (248) = a Traced area} Clearly, when r = R, V +0. (0) o= co (Bq, 249). In this case, 9% = $Y at the point r = R. Therefore, o(@) = (as) {Hoe +0? arecase)-*”*(2r ~ 2acosé) r + £08 (apn? —2oca)** (Sar—toeut)} = “tf (R? + a? — 2Ra.c088)-*/?(R - ac0s6) + (R? + a? —2Racos6)*? (G -acoee) } = Lee +08 — aRacosey*? [r-ecora- + acon] = [at eyes toca =f oda= 4 vi a? ~ 2Racos6)-¥*R? sin db de = 2Racos6)"*/? = yt ote [Fee +e ~ aRacosey-*™) = t@-R) pero . aeoal 2a VECO Re” Tie ae But a > R (else q would be inside), so VP ya! — 2Ra =a~R. q 21 1 1 =i = dem [ta- wal-2 Hullo W)~ (0+ R= 3-20) | “4 CHAPTER 3. SPECIAL TECHNIQUES (@) The force on g, due to the sphere, is the same as the force of the image charge q to wit: a i(k ‘) 1 eRe ines (a Bt = neg \~ a ) (a RBJay? ~~ Greg (a? = RAF ‘To bring q in from infinity to a, then, we do work: ° wares Fro He? — a-fe aesml Problem 3.8 Place a second image charge, q", at the center of the sphere; this will not alter the fact that: the sphere is an equipotential, a= rel wat merely inerease that potential from zero to Vo = zh-2 —— but merely P eae rF 3 For a neutral sphere, q! +4" = 0. PF fe -t)\- mw (141, wat (+ wp) & Cota) qq’ (20-8) _ g(~Rgla) (R?/a)(2a~ R? fa) qe ara @(a— Maye 2 ny ee R) ir a) (=P (Drop the minus sign, because the problem asks for the force of attraction.) - Problem 5.9 (a) Image problem: A above, ~A below. Potential was found in Prob, 2.47 2» Rape ve ese fa= apg — ae Aon (Ets) ira" FFG stom OV (b)a= ORT Here ‘oni aes evaluated at z i ow) = -og: (Sy = ae Check: Total charge induced on a strip of width U parallel to the y ais _ nd Fa Dd PL Hy]? fms = w= (| A. Therefore Aing = —A, as it should be. 45 ‘roblem 3.10 ‘The image configuration is as shown. 4 ae) 1 Cee aes eae pan Verare@ ree aes For this to work, [@ must be and integer divisor of 180°.] Thus 180°, 90°,'60", 45°, etc., are OK, but no others. It works for 45°, say, with the charges as shown, (Note the strategy: to make the 2 axis an equipotential (V = 0), you place the image charge (1) in the reflection point. To make the 445° line an equipotential, you place charge (2) at the image point. But that screws up the z axis, so you must now insert image (3) to balance (2). Moreover, to make the 45° line V = 0 you also need (4), to balance (1). But now, to restore the © axis to V = 0 you need (5) to balance (4), and s0 on. ‘The reason this doesn’t work for arbitrary angles is that you are even- tually forced to place an image charge within the original region of interest, and that’s not allowed—all images must go outside the re- A gion, or you're no longer dealing with the same problem at all.) me sia) hy 8 docan't work for @= 195" ("Sroblem 3.11 ope [eter] ea in| EAE AL | | where a? = yo? — R* > [a 0! [ena ty w wo Gaon (2), [i ES] From Prob. 2.47 (with yo ~+ 4) and acoth(2neoVo/§) =a { acsch(2neaVo/d) = }+ (ai Problem 3.12 View) = oe e-™/* sin(nry/a) (Bq. 3.30), where Cy = 2 2 frsensnoraros (Ea. 3.34) 4M, for 0 Sle e-rsTe ~ Sink(nz/a) Therefore ) and [ve y 0, with boundary conditions @ V(z,0) =, (i) V(z,o) =0, (ii) V,y) =o, (iv) V(b,u) = Voly), Asin Ex. 2.4, separation of variables yields a V(z,y) = (Ae* + Be-™*) (C'sin ky + Dcos ky). Here ()=> D=O, (ii) B =~A, (i) he ie an integer multiple of x V(z,y) = AC ("7/2 — e"**/*) sin(nxy/a) = (BAC) sinh(nrz/a) sin(nzy /a). But (24C) is a constant, and the most gencral linear combination of separable solutions consistent with (i, (a), Gi) is ley 2c . ¥(2,y) = D> Ca sinh(naz/a) sin(nmy/0). aie _ 1k remains to determine the coefficients C, so as to fit boundary condition (Iv): Xen sinh(nnd a) singny/e) = Vola). Fourer's trek => Cn sinh(nand/a) = 2 f Volu) sin(nzy /a) dy. a Therefore aaa | Yolusin(nu/a) dy 48 CHAPTER 3. SPECIAL TECHNIQUES 2 -— 2% __ { % ifniseven, (©) Cn = seinianbay'® | Simlnul®) dy = Samaj {. if nis odd. } Problem 3.15 ‘Same format as Ex. 3.5, only the boundary conditions are; when when when when when when This time we want sinusoidal functions in x and y, exponential in z: X(z) = Asin(kz) + Boos(kz), ¥(y) = Csin(ly) + Deos(ty), 22) = BevPV* 4 Ge VPM (3 B= (i)> k= ne/a; (i) D = 0; (iv) 1 = maa; (v)> E+G = 0. Therefore (2) = 2B sinh(x Vn? + mP2/a) Putting this all together, and combining the constants, we have: Veewz) = Cam Sin(nix/a) sin(mry/a) sinh(w Vn? + mz /a) I remains to evaluate the constants Cy,m, by imposing boundary condition (vi): w=>>d {en sn sinh(x Vie + m?)] sin(ninx/a) sin(may/a). According to Eqs. 3.50 and 3.51 Cams (VF) = (2)'v5 f [imtoctorstmns/daeds= {Sine oy nn ‘Therefore 16h SE snore) sinonry fy 28h LEELA) sinh (Vn? me) V(ey2) net Sin maha, 49 Problem 3.16 Pale) Sey = Sae@-y' ote -1)* [(2? = 1)" +22 (2? -1) 22 2] = thle ye —1+42")] gy lle? 1) Ge? —1)) = § [ae (a4 — 1) + (@? 1) 102] a! — 24 Sa — 52) = 5 (102° — 62] where Ps (c086) = }cosd (Se0s?@ ~ 3). a 5 [-8ine (Scos*@~ 8) +cos6(10os6(—aind)] = —1sin0 (Scos*d ~ 3+ 10cos*o) 3 2 = —Fsind (Sc0s? #1). 5 (sno) = $795 bin? # (cost ~1)] = 3 fasind cosd(5e0s*— 1) + sin? #(-10cos@sin6)] = ~8sindcos0 [5cos?#— 1 ~ Sin? 4] aaa (sno) = ~8e0s0 (Seas? —1~ 5 (1 -c0s*6)] = ~Bcas0 (10:08 9 ~ 6) = ~8-4- fost (5eas!@—3) = -1 +). ged [Pcerrcayae= [eh (24-32) de =} (os 2) p0-141-n=04 Problem 337 (2) Inside: V(6,8) = J Aur'R2(cos6) (Bq. 3.66) where = AS FEY [ vileri(eoso)sino a (Ba. 3.69). In this case Vo(@) = Vo. comes outside the integral, so p= CAM F rycsoy nee : 50 CHAPTER 3. SPECIAL TECHNIQUES But Ps(cos4) = 1, s0 the integral can be written | Pe(c0s8)P(c080) sin do ts eal (Bq. 3.68). ‘Therefore A AFD ALK, ol Plugging this into the general form: V0) = o2P(cos8) =[Ve.] ‘The potenti content drat he shee Outside: V(r,8) = rhea) (Eq. 3.72), where a = Sedge : Vo(®)Pi(cos8) sind (Eq, 3.73) — ED way F ee ino = Gg feanineann[ Oy HEY R],, 1 Therefore] ¥(r,0) (Ge. equals Wo at r= Fy then falls of ike 4) © c Sat ncos6), forr< R (Eq. 3:78) Von=) ; Yo FE Pleost), forr> R (Ea. 8.19) where - = R14, (Eq. 3.81) and | A= oe + {tore sno (Ba. 3.84) - otto _—e salen [ now sind dO = | ten, too }. ‘Therefore a V(r.8) = ol terms of the total charge Q = 4x00, 1@ eG rer Veeo)= Bef, roe Problem 3.18 Vo(8) = kc0s(38) = k [4cos? @ — 3086] = k[aPs(cos#) + BP, (cos 8)} (C know that any 3 order polynomial can be expressed as a linear combination of the first four Legendre polynomial; in this case, since the a is odd, Tonly need P, and Ps.) cos? #~ e050 = a [E (5cos!o— cost) | + e058 = Senso + (p- § 088, So 8 42 eand, -se9- Therefore Wo@ = Now . VAir'F,(cos 6), forr << R (Ba, 3.66) Vena) S & PIC (cos0), forr> R (Ea. 3.71) sere A = FAD [yA eos0)sinddd (Eq 3.69) . oe | J rxcntyntcont) sind afr cnapiounsneah _ kD, 2 bing 2D gt st 85a 364) si/sm, itt=3 = {eee HILT } eee otherwise Therefore 3 sks aa eel YO) = Far (ost) + sper Please) =| [() Fes ~3(F) Blew) | or £ {8 (Z)° 5 [cose ~3coss) ~3(4 oma o> ono {a (E)' Beata 3-3} 52 CHAPTER 3. SPECIAL TECHNIQUES (for r ¢ R). Meanwhile, B, = 4,R%*1 (Bq. 3.81—this follows from the continuity of V at 2). Therefore pa { BRIS, if t= t= | ~3kR/5, ift=1 x0 otherwise). SRE T py(coss) + ERE A » (cos) AG (8) acoso -3 Vin,0) 4 ) eate 3] By (for r > R). Finally, using Eq. 3,83: (8) = co (214 DAR P(c0s6) = [VAP + TAR? Ps} (_3k 8k ) pa, cok ae = of (-#)4+7(S) #9] =| G97 (one) + 8670089) = 2 | _ gcse + © (5 c0s'@ -3c056)| = 8 cosal— cos? — = [90 0+ 3 (Beast os. 0) = SF ona 9+ 28-5 cos? @— 28-3] wok = [Sh ose (roc - Problem 3.19 ~ 7 i 41 Use Ba, 3:83: 0(8) = «0 ) (H+) AiR! (C080). But Eq, 8.69 says: Ay = 2 [voor (cose) sino ro d Pucting th together: [ v(F(cos8)sin0d0. ged 216) = 2. 5-214 17C,R(eon0), with C Problem 3.20 Set V =0 on the equatorial plane, far from the sphere. Then the potential is the same as Ex. 3.8 plus the potential of a uniformly charged spherical shell: 53. Problem 3.21 2, V6.8) = DBM leos6) (> 2), 0 V0) = FoR) =F = = , oR? ont Comparing like powers of r, I see that Bo = 97, By =0, Ba = fg... Therefore ve) = EE Breane], Elz (2) eaves. (0) V(0,0) = So Avr!P(c0s0) (r < R), Sn the northern hemispere, 0 < 0 < x/2, (for r > R) Vine) = Sant = 2 [VERE] _ i Since r < Rin this region, Vr? R? = RYTF TRY? = alt + }oye)~ fe/my'+ | Therefore Yas! Comparing like powers: Ay = 5°, Ay = eo « ey = yee of. i... 7 4 Yon = £ i = rPr(cone) + Fa Ps(oos) +...|, \ (Gor < R, northern hemisphere) on 1 F)’ (Beosto~1) +. Un the southern hemisphere we'll have to go for 6 =x, using P-L) = (+1)! Von) = So(-v'aet = 2 [Vite 2 25 z CHAPTER 3, SPECIAL TECHNIQUES (C put an overbar on Ay to distinguish it from the northern Ay). ‘The only difference is the sign of 2 Ay = +(0/260), Ao = Ao, Az = Ar. So: (for r < R, southern hemisphere). & [P+ rr cosa + sr? Paloose) + }. Al | [+ (f) e045 (F Boot -1)+ Problem 3.22 SAurA(cose), (F< R) (Ba. 3.78), von =d y Pir (oos6), (r > R) (Eq. 3.79), where By = AvP! (Eq. 3.81) and 1 ha aa | HMA cot) sinoas (Eq, 4) =. { Pnvonsinoas- [rcomaea} een th 5 adi {fae near} Now P)(—2) = (-1)!Fi(2), since Fi(2) is even, for even 1, and odd, for odd !. ‘Therefore Jrterae fnc-sa-2)= (at freee, and hence 1 6 if Lis even Ave Ser bh C0) [ mae = aiiis f Plerde, itis oad 55 Therefore and ‘Thus war 7 vanel i [pico - ; EOD) cont ot [a (cos) = Problem 2.23 ~ - “(+ Look for solutions of the form V(s, 4) = S(s)®(¢): Multiply by 5 and divide by V = $: _ Stas) ts Since the first term involves s only, and the second ¢ only, each is a constant: iy AEE Ca, with +Cs 5a (@) 56 CHAPTER 3. SPECIAL TECHNIQUES Now C, must be negative (else we get exponentials for , which do not return to their original value—as geometrically they must when 9 is increased by 2r). Moreover, since #(4 + 2x) = @(¢), k must be an integer: k= 0,1,2,3,... (negative integers are just repeats, but k= 0 must be included, since # = A (a constant) is OK) a (as of (88) HS can beset § , provided n is chosen right: d rise"! = nts" = BS 3 n= th. ons*-") = ned (ot Evidently the general solution is S(s) = Cs* + Ds™#, unless k = 0, in which case we have only one solution to a second-order equation—namely, S = constant. ‘So we must treat k = 0 separately. One solution is a constant—but what’s the other? Go back to the differential equation for S, and put in k = 0: 4 (as as as_c ds : of (8) = 05 0 = constant = 0+ B= 2 as = 0% = 5=Cins+D (another constant So the second solution in this case is Ins. [How about 4? That too reduces to a single solution, # = A, in the case k= 0. What's the second solution here? Well, putting k = 0 into the @ equation: ee ab oe 02 = constant = ag dg But a term of the form B¢ is unacceptable, since it does not return to its initial value when ¢ is augmented by 2r.] Conclusion: ‘The general solution with cylindrical symmetry is =O BO+A V(8,9) = a9 + bolns + > [s*(az coskd + by sin kd) + 97*(cy coskd + dy sin ‘| Yes: the potential of a line charge goes like In s, which is included. Problem 3.24 ~— - Picking V =O0on the yz plane, with Bo in the x direction, we have (Eq. 3.74): rr, () V=0, when s = R, (i) V4 Bor = ~Foscosd, for s>> R. Bvidently ay = 0, and ag © except for V6.9) = (19+ S) cose. (i)=> e = —a,R*; (ii)-+ a, = —E. Therefore VOs,9) (-os+ ) ond, or }¥is,9) Y- 87 — Problem 3.25 we i SS Inside: V(s,6) = a9 + 5 s* (ax coskd + by sin kd). (In this region Ins and s~# are no good—they blow up ats=0) . ea , Outside: V(0,9) =a + 4 (e,cosko + dh sink9). (Here Ins and st are no good ats -+ oo) ‘Thus Bvidently a Rao Ue ees. (Fe +H) ‘Also, V is continuous at 5 = 20+ Rb sin Sp = d+ Feds sin 5p. So a9 = dy (might as well choose both 2270}; Ribs = R-Sdy, or dy = Rb is - 4g) = WcoR*s; bs = — 2s dy = 2 Combining these results: a = Sep (R*bs + Rtbs) = leo R*bs; bs TocoRt* % = 7pc5 Therefore Vie) =28 mf afm isi 10 | RY/s*, for s > R. hier 5.26 — = Monopole term: fr r) sind]? sin 8 dra Q= [oa =f [5 a(R 2) sing] sind ad Bur the r intagcal is [(e-mar= (Rr= rN = R=0. SoQ=0. [romepdr= Er (ome) | But the @ integral is 8° ecosba8 = 2 So the dipole contribution is likewise zero. Quadrupole term: [o(Gove-3) etre ben f [2 nto [Jea—29 509] Dipole term: (R= 2nyaind] ? sind trees in dr do. 58 CHAPTER 3. SPECIAL TECHNIQUES 1 integral a 2 ROR Re Punanér= (5. RRR 6 integral: J ores) saroan=2 fewtoae sf atoan Jo (ttn ID Balnt a a « 3 ® =96)-8(8) -*(-9)=-§ @ integral: The whole integral is: Problem 3.27 p= (39a ~ga)2 + (—2ga ~ 2q(~a)) § = 2¢a. Therefore and p-f = 2qa2-F = 2acos8, 50 Problem 5.28 (a) By symmetry, p is clearly in the z direction: p = p% p= [ zpdr => [ z0da. p= [(Reaseyecoseyn*sinadedp=2nReh [ cos osinaad = annie (-25°*) 3 lo = entay —(cay = SeRE ak = Jerig-(ol= 24, ©) 1_4eR*k cost eS (Dipole.) This is also the exact potential. Conclusion: all multiple moments of this distribution (except the dipole) are (7 exactly 22r0. Problem 3.29 Using Eq. 3.94 with 1! = d/2: But Pa(-2) = (-1)"Pa(z), so 1 aye vegeo(2-})-25 ae(¢ y [Pq(e0s8) — Pa(—c0s6)) = x (2) aacora Therefore 2 seat —— Van = GEES A (cost) = SEP, white [Vous 0 yd Yous Gt, (Z)' tee) = 22S} (scot o Sess) =| 2, (sent 0 Sed). Problem 330 ©) 00=[%] 6) p= [ink] wv =a; [2428] = ©) Q=[Ra] Gi) p= [aaa] iy v= (@Q=[2a] @)p=[Baas,] wiv = Problem 3.31 (@) Tois point is at =a, @= §, 6=0,s0B= ~P 6 =P (8); Fal ae Gregg? oa Ameo’ (b) Here r = a, @ = 0,80 B= 72-524) = ieaat (0) We @{¥(0,0,a) - V(a,0,0)] = Problem 3.32 Q = ~G, 80 Vineno = ane ees 60 CHAPTER 3. SPECIAL TECHNIQUES Problem 3.33 . P= (p-#)# + (p-6) 6 = peos0# — psind 6 (Fig. 3.36). So 3(p-#)# ~ p = 3pcosO# — peosd# + psind 2pcos0F + psinB 8. So Eq. 3.104 = Eq, 3.103. ¥ Problem 3.34 At height 2 above the plane, the force on q is given by Eq. 3.12: F dy Multiply by o= 4 ot Lg@_ i @s bs ae ae Alz, where A= Ay constant. ? = Teneo But v = 0 when 2 = d, so constant = —A/d, and hence v? = BA fa Vav [Ra TA 7 eal ae ‘This integral can also be integrated directly. Let = u2; dr = 2udu. a1 NEP (G) va ‘Therefore Problem 3.35 webs fs +. +h ts ae ‘The image configuration is shown in the figure; the positive image charge forces cancel in pairs. The net force of the negative image charges is: lof 4 1 1 P= ire? {sea * Rovde-ap * fatde-ap* ot } "G2 Gat tay” Gat Rep vale ut mot | later tera} 1 1 ¢ When a+ 00 (ie. @ > 2) only the 2 term survives: F = 7-755 ¥ (same as for only one plane— Eq. 3.12). When 2 = a/2, we leper cone wert) [tart aap * wae] Problem 3.36 Following Prob. 2.47, we place image line charges —A at y = 6 and +A at y= axis, z vertical). malh() = anf =D (here y is the horizontal ato fa-b\?_ (a+b)? Spt (st) = (GR) see +(2)] =a" [y+ a)? + 27}{(y — 0)? + 27) (ya)? 2 +? +=] =a vo wy Problem 3.37 Since the configuration is azimuthally symmetric, V(r,0) = J (a + +a) Fi(cos8). a)r>b: acrcb: Ay =0 forall, since V0 at 00. Therefore V(r,6) = nen vo. = (ce! + #) Pi(cosd). rca: V(r6)= We need to determine B1,Ci,D1, and Vo, ‘To do this, invoke boundary conditions as follows: (i) V is EZ 42) at b. continuous at a, (i) V is continuous at b, (ii) A (5) = ¥ (a + gi) Ateosay, Fra cw' + PL [a= MaxD.]a Bar Fi(cos®) = sce 02d (cit + =) P,(cos8) = wf ea Putting (2) into (1) gives By = Gi) > TBA + Mhz Fcose) - _+)), +), wre MC, 10, Ds = aVo~aCo Ca! ++ PL =0, i140, it 1=0; Coa? + = = Vo, BG aC, 140, Bo = bCo+ aV4 — aC Therefore Boe Sa . By = (b~ a)00-+ ao 0 P,(c0s 6). So D (ciate + GP) reosey = cue? g =0, fl 41; 1 (cat +D fee) 62 CHAPTER 3. SPECIAL TECHNIQUES =(L+ 1B, = 10"! + (14 1)Dr <0; (C4 1)(B = Dy) = = 181. Bede (4 DF G+ 2 -D) =# ‘Therefore (1+ 1)(By ~ Dy) + oC; c+ p(B -D)=* for 1 1, (@) Plug (2) and (1) into (3): For 1 0 or 1: (le) (08 — a) CHC) 40, = 0; LENG FOC, = 0; (2L41)C; = 0+ C, ‘Therefore (1) and (2) [Br == Di = Ofer > 1] Fort =k G+ Ze) G+eC] = & +20; Sif] B= (-o') 0 = [= Paka! For! = 0: Bo—Do = 0 => Bo = Do = (b—a)Co+aVo = aVo—cCo, 90 bCy = 0 = [Co = 0; Do = aVy = Bo. @ ek 7 0 + Dy oneuson: [ied = 8 5 PATE asp rot venir £ (6-8 6) =~ H Wey Econo . Ye 4 2 = a1 1 Q__1 draat a= / oda = “2 4na? = [4rocale = Qu] At larger: Vins S42 EG — Problem 3.38 a Use multipole expansion (Eq. 3.95): pdr > Ade = 5 de, and r! — =: Ved = GY as | Paeosey ae tre The egal Q ea) ant 2artt Brose f2ree= Brean ZA = Brean tornern, wt or noi ‘Therefore Problem 8.39 Use separation of variables in cylindrical coordinates (Prob. 3.23): V (6,4) = ao + bolns + 3 [2*(ay cos kp + by sin kd) + s7*(ox cos kd + de sin k6)] 6 SER: V(54) = DE sax coskd + bysin kd) (Ins and s* blow up at s = 0); 5>R: V(s,d) = De s"H(ex coskd + dy sinkd) (Ins and s* blow up as 8 -+ 00). (We may as well pick constants so V -+ 0 as s+ 00, and hence ay = 0.) Continuity at s = R > SO RMaxcoskp + by sinkd) = J R-*(cecosks + dysinkd), soc, = Ray, de = Feby. Eq. 2.36 says: 1 wv ~ Bs ou -. Therefore 98 In 3 pati (or cos + dy sind) ~ J AR "(a, cosh + be sinkd) = -20, 1, 7 af of (0 or Band hence |e = ~2-6%() seo. Seo @) 1 Ewe = BT f AS'(e) dr Problem 3.43 ee (a) F= [(VVj)- (VV) dr. But V - (Vi Va) = (VV): (VWs) +4 (V7V4), 80 [o-worvnar- [reve = f wer) -dae & J vipade fs @ But the surface integral is over a huge sphere “at infinity”, where Vj and V2 0. So 1 z 7 Vipedr. By the same argument, with 1 and 2 reversed, = 2 | Vapi dr. So _ Vipadr = _ Vapidr. ged 66 CHAPTER 3. SPECIAL TECHNIQUES Situation (1) : Q ©) { [, prdr =Q5 Qs = fyprdr =O; Vie = Vos. Situation (2): Qa Ipprdr =0; Qs = J, pedr =Q; Vaa = Vow SViprdt = Via Spe dr + Vie Spr dr = VasQ. J Vop. dt = Vea J, pr dr + Von fr dr = Vie. Green’s reciprocity theorem says QVgs = QVia, 50 Var = Via- ged Problem 3.44 (a) Situation (1): actual. Situation (2): right plate at Vo, left plate at V = 0, no charge at =. fiat [Ynar But Vi, = Vp, =0 and Qe, =0,50 J Vipadr =0 [Yea But Vi, = 0 Qay =a) Ven = Vos Qn = Qa, and Ve, = Vo(e/d). S00 = Vo(2/d)q + VoQa, and hence Vis Qty + Voy Qes + VexQrar VisQt, + VarQz, + Vis Qn Qa = ~a2/d. Situation (1): actual. Situation (2): left plate at Vp, right plate at V = 0, no charge at 2. [Yee =0 [vn dr = VigQn + VenQey +V¥eQr, = VoQi + Vex + 0. But Vez = Vo (1- 2), 0 a (1 = 2/d). (b) Situation (1): actual. Situation (2): inner sphere at Vs, outer sphere at nero, no charge at r [Yea dr = Vases + VeiOn + Vie So J Vipzdr =0. J Vierdr = Vases + Van + Vinh = Qo¥o + Wr +0 But V, aAsB ~a¥o/(b— a); BU: the potential at r in configuration 2: V(r) = A+ B/r, with V(a) = Vo > A+ B/a = Yo, or a¥, and V(b) = 0 > A+ B/d = 0, or bA+B = 0. Subtract: (b-a)A = -aly > A= 4) = Vo = BEG = B = abVo/(b— a). So V(r) = (fd; (f -1)- Therefore 67 (Now let Situation (2) be: inner sphere at zero, outer at Vo, no charge at r. 7 [Minar = ‘This time jsplaystyleV(r) = A+ 2 with V(a) = 0 A+ B/a=0; V(W) = V2 A+B/b=Vo, 90 vie) = qt (1-8), Theirs aE (1-2) + Q6¥0=0; [Qs=— gH (1-2) = [ Vopr = Vos + Vex + Vinh, =O aes + QV (b=a) Problem 3.45 (a) -eP Eras} par! aH ol = Stns Stiby = Dh FF 8 Lis. | La Vout = ge [5 (remot 1 ") pe --73 (b) Because 2? = 9? = (0/2)? for all four charges, Qez Because z = 0 for all four charges, Q., = —(v2a/2)(q This leaves only Gey = Ore = 3[(5) GG) a+) Ca) 0+ 5) (G) + (5) (-$) of ~Biee + Pa(cos@!)pdr" (the n = 2 term in Eq. 3.95). av = [3(0/2}? ~ (v20/2)] = a440) =O and Qez = Que = -ata) Quy © co Cae / [Btri = di)(ry ~ 44) - (F - 4°65] pdr (11 drop the primes, for simplicity.) a flores = 76] pdr a; [ rypdr~ 24, [ npar + saad f pdr-+24- f xpdrb — 64; [ par = Qi — 3(dip; + djp,) + 3didjQ + 265d «p - d°5yQ. Soif p=0 and Q=0 then @, (d) Bag. 3.95 with n = 5 ig =Qij- aed Vea ak [Oe v*Pueose pdr’; Pycos0) = 3 (Seo 0 Bconé) Define the “octopole moment” as 68 CHAPTER 3. SPECIAL TECHNIQUES “rae ((é-2)oe(E-2} a = Vitta?— Braces, n Vr? +a? + 2ras0s8, VP +i —Irbc0s8, VEE irbeos8 ar Psgntingacn Be 0; (2-2) » Bent oe mano 0+ 2 ‘cos@ (here we want b L (4) cos(k'#) ofr (We)ae { tone But 0, ifkFk * cos(kg) cos(k'd) dd = { n (S80 Brag = Vor/2 = ap = Vo/45 naxR* = (2Vo/k) sin(kr/4) = ax = (2Va/rkR*) sin(kx/4) (k # 0); hence 7 out] ven =vf i=? 5 sinter) Using Eq. 2.49, and noting that in this case # = aly? oD ptt cosh, = Me MRS sen co ol8) =e ‘We want the net (line) charge on the segment opposite to Vo (~ <4 < ~3x/4 and 3n/4 < $ <7): sate A sine) Hee -0s(kp) dé A= fonde=2n a(é) ap . oh $ Es in(hx/4) [fae tds Yo $ sin) sinh) Sa(ee/a) anlSkx/A) product rc 1/vz V2 1 Q ai " aa f Ouch! What went wrong? The problem is that the series 32(1/k) is divergent, so tle “subtraction” 0 ~ 00 is suspect. One way to avoid this is to go back to V(s,¢), calculate o(OV/0s) at ¢ # R, and save the limit, rc 2 CHAPTER 3. SPECIAL TECHNIQUES 3+ Runtil the end: ov date F sin(kr/4) ks! ar OF FE cost) o(¢,s) = dae Sat + sin(kn/4) cos(k@) (where 2 = 3/R-> 1 a the end} Ae) o(6,s)Ras sea [1 (2, = ME (erg _2e0¥o [+2 2 +E+ FE ae L But (see math tables) :in (F a4(pu(t Now consider the pendulum: F = —mg%~T, where T ~ mgcos¢ = mu?/I and (by conservation of energy) mal cos = (1/2}mv? = v? = 2glcos¢ (assuming it started from rest at = 90°, as stipulated). But cos $ = ~ cos8, so T = mg(—cos6) + (1n/l)(—2glcas8) = ~3mgecos8, and hence F = ~mg(cos0# ~ sin46) + 3mgcosd# = mg(2os0# + sind) This total force is such as to keep the pendulum on a circular arc, and itis identical to the force on g in the field of a dipole, with mg + gp/4reol*. Evidently q also executes semicircular motion, as though it were on @ tether of fixed length !. Chapter 4 Electrostatic Fields in Matter S210 Tale 4.1: a/Ara = 086% 10°% a0 = An(8.85x10-"9(066%10-) = B= T34x10-", p= aE =ed = d=aB/e = (7.34 x 10-)(5 x 10°)/(1.6 x 107) = 2.29 x 107 dR = (2.29 x 10-*9)/(0.5 x 10- Rex/ox = (0.5 x 10-"9)(1.6 x 10-!9)(10-)/(7.34 x 10-**) = [10° V. 46% 10-*] To ions, ey d= R. Then R= abe = aV/ez 9 V = Problem 42 First find the field, at radius r, using Gauss? law: f Beda = 2 Quy, or = ghd Qene [Note: Qene(r > 00) = q.] So the field of the electron cioud is By = => ft eile (Q 4254 28))- The proton will be shifted from r = 0 to the point d where Ey = E (the external field): = fret (1424 42% e- hi e 1425425 Expanding in powers of (d/a): ome a 1-(2)4 GZ) iG) sontatea()'-$( soem (uaberf) « = 3 (1-982) -4() +) (228) = y-y-2f- $obeedod- hbase = (2) + ttre tame. 2 ™ CHAPTER 4, ELECTROSTATIC FIELDS IN MATTER 1 4d 1 1: R, same as if all charge at center; but Qrot r 76 CHAPTER 4, ELECTROSTATIC FIELDS IN MATTER. Problem 4.11 Ps~ 0; 05 = Pei P points away from), at the other—the one +P (plus sign at one end—the one P points toward; minus si () £> a. Then the ends look like point charges, and the whole thing is like a physical dipole, of length L and charge Pra®. See Fig. (a). (i) 1 E = (p/2a)s. For two such cylinders, one plus and one minus, the net field (inside) is B = Ey ¥ B_ = (p/2e0) (sy ~s-). But 84 ~8. = —d, so E ={—pd/(2eo),] where d is the vector from the negative axis to positive axis, In this case the total dipole moment of a chunk of length € is P (ra) = (pra*¢) d. So pd = P, and [B= —P/Qeah] for s b, Qenc ) (ana?) + f° (GB) Anvtar = —arka ~ Ank(e ~a) 0 (Prob, 4.14), so [B=0.] Ankr; so[B = —(k/eor) Gauss's law = B= 7h! Fora O) 0 > D =O everywhere. D = @B +P =0- B= (~I/)P, 90 —(k/eor)# (for a 8 CHAPTER 4, ELECTROSTATIC FIELDS IN MATTER Problem 4.17 6 @® etc) (field of two circular plates) _ (same as E outside, but lines continuous, since V-D = 0) Problem 4.18 (a) Apply JD -da = Qj... to the gaussian surface shown, DA = oA = [D=o.] (Note: D = 0 inside the ‘metal plate.) This is true in both slabs; D points down, ed: /ea in slab 2. But © = cote, $0 €1 = 205 € = (0) D= cB > B= o/c in slab 1, 2a/3eo. (©) P= @xeB, 80 P = coxed/ coer) = (e/en)0i Xe = e139 P=(l- Go. [P= 9/2,|[R=0/3 (a) V = Bia + Eya = (a/6e0)(3 + 4) = [Toa/6eo. Ey =o), P, at bottom of slab (1) = 2/2, tS Pat top of slab (1) total surface charge above: o ~ (0/2) = 0/2, total surface charge below: (2/2) ~ (0/3) + (0/3) ~ comand t: { total surface charge above: o — (0/2) + (« ie ~ (0/3) = 20/3, _ 2a a a ee ce Soe Gi oe lor jamie ————————— oe — @® Jo aS" Problem 4.19 With no dielectric, Co = Aeo/d (Eq. 2.54) In configuration (a), with +7 on upper plate, a on lower, D = a between the plates. E = o/eo (in ait) and E = o/¢ (in dielectric), So V = f= 25 (144) G. w= Geud “f (ie) > Im configuration (b), wth potential diference V: E = V/d, s0 0 = co = «aV/d (In air oxeV/d (at top surface of dielectric) oy =a (I+ xe)/d= coeeV/a (on man Sa3 (A to/4) <2 (ol ruta) Oa] It the 2 axis points down: I _P (op sats) I 6 (op pa) | i ° 2 cine | Ww dicocnic mraz} rae Oar o 0 =o] (oy deectrie eanaee | ee ee ae | Probiem 4.20 [Deda pRB 30? > E .. > DAnr? = phar? => D = Lor > E = (pr/30)8, for r < Rj D4nr? (oR? /3eor?) , for r > R. Og ee gee ee ee oem Problem 4.21 Let Q be the charge on a length £ of the inner conductor. oe eee asl! reat i LE) E+f GE) S-Sahh(2) +2] a fdvaa = Dinst=Q4D (a a), subject to the boundary conditions @) Ye (i) He ii) Val > EgScosd for s >a From Prob. 3.23 (invoking boundary condition (ii) Vials) = So s*(an coskd-+ businkd), Vour(s.6) = 80 CHAPTER 4. ELECTROSTATIC FIELDS IN MATTER (eliminated the constant terms by setting V = 0 on the y plane.) Condition (i) says Laker cosh + besin kg) = Fyscosd-+ J a-*(e,cosko + de sin kp), while (i) says 6 Xo ha (a, c05 kG + busin kd) = —By 008 6 ~ J ka“**(o, cos ke + de sine) Evidently by = dy = 0 for all k, ax = ¢k =O unless k= 1, whereas for k= 1 am =-Rata'e, €-0, =—-Fy—a 7, Solving for a, B eae . eterna reap tetera) and hence Eia(s,¢) = ~ 5 aH | 8 te spherical case (Ex. 4.7), the fel inside is uniform Problem 4.23, Po = corel: Bi = -gePo = -SEo; Pr = oxeBs = 2 = (-X2)" By Evidently Ey = ( %)" Bo, 80 nensn snr [E ‘The geomettic series can be summed explicitly which agrees with Eq. 4.49, (Curiously, this method formally requires that ye < 3 (else the infinite series diverges), yet the resul is subject to no such restriction, since we can also get it by the method of Ex. 4.7] Problem 4.24 pana) Potentials Voue(r.8) = —Korcos6 +52 Bir Pi(cos®), (>); Vnoa(rs8) = Y(Aut + Bir) P(cose), — (a Benen + FP Moos = TP (At! ge) Ales; @) = 6d [ise = (+ aga] Pi(cos8) = ~Fyc0s8 — SU+ IP Pos); i) > Aas Poo B= Port #1: Bea (a 2) = maa omy, j) « [aw earn A) = = a.=-4[(4) oer ya] Ae Por ob? = A12 (8° — a8); @ w «(4 = -2B, — Fob? = eA, (b* + 203). So -3Eqb* = A, [2(0? — a) +6, (8 +20")]; Ar = GEG r = oem (rom — amare azar ( ame 3 2a" Be) = Ws =| amine adr s wan (1H) et Probiem 4.25 7 ‘There are four charges involved: (3) g, (ii) polarization charge surrounding q, (ii) surface charge (04) on the top surface of the lower dielectric, (jv) surface charge (of) on the lower surface of the upper dielectric. In view of Eq. 4.39, the bound charge dy = ~4(x,/(1 + x2), 80 the total (point) charge at (0,0,d) is = 94% = af +X2) = Vel. As in Ex. 48, - ladle _ % 8) (here 94 = Po = +P, = Go = cx [a Gaal 2 4] (here oy = PsA = +P, = eoxeEs); = gy [Ladle 8) here op =P, = me (b) o) = coe og +ayt 2 | (here o» = ox. Es)- Solve for 04, 04: frst divide by xe and x, (respectively) and subtract: pa, [te 4 2 ale” oeinn [Ben oe] o_o 1 ade. Xe Xe Ean! 82 CHAPTER 4. ELECTROSTATIC FIELDS IN MATTER, Plug this into (a) and solve for 0, using ¢ = 1 + x! Xe Xe + xe)/2] Foe + xd), 80 a - v{2— 1 pe edtlee so oS MV Gr Ge eit Oe txO/ eee ey? fs’ exe | (r2 4a?) 2 (T+ (Xe + X2)/' ‘The fotal bound surface charge is 0; = 0540}, few) atte archi (which vanishes, asi should, when XL = Xe). ‘The total bound charge is (compare Eq. 4.51) sear, snce $ +q.2 2 Problem 4.26 Brom Be 45: 0, (r ) _ pfoeen Soil! f harwed (dal Slt (2)f = galamaG-3)* Cr Problem 4.27 Using Eq, 4.55: W = f E%dr. From Ex. 4.2 and Eq. 3.103, RPh (r R) eo (P)?4 og, PAR Wren = (é) a Te co (RP)? fa A Woe = 2(%) [Folds 8+ sin) sina das 5 ee x(RP) ae ole . 2n revs ingae [5 Ten (~ e088 ~ 00°63 (-#) i" _ on ws) 4nRBP? © 80 (SRB) Be We = [2RP 7 This is the corvect electrostatic energy of the configuration, but it is not the “total work necessary to assemble the system,” because it leaves out the mechanical energy involved in polatizing the molecules. = }JD-Edr. For r < R, D = eB, sa this contribution is the same as before Por <8, D = @E+P = -}P +P = 2P = ~20B, so }D-E = ~2$E7, and this contribution is now (2) (32228) = —$¢ F*, exactly cancelling the exterior term, Conclusion: [Woy = 6.] This is not surprising, since the derivation in Sect. 4.4.3 calculates the work done on the free charge, and in this problem there is no free charge in sight. Since this is a nonlinear dielectric, however, the result cannot be interpreted as the “work necessary to ascemble the configuration” —the latter would depend entirely on how you assemble it, Problem 4.28 First find the capacitance, as a function of fr Air part: B= 2, 9 V aii Oil part: D= BL + B= BL Vv = Bin(o/ay, } Q=NK+ ACh) = erMh ANAC = Mer — Wht C= Axe +O), where € is the total height, Ga Da HO, 9 Oech 40 V > Bln(bja) 7 = 2" In(o7a) ” ‘The net upward force is given by Eq. 4.64: F = £V24E = L244, } peta The gravitational force doun is F = mg = px(¥? ~ a)gh = ob a?)gin(b/a)” 4 CHAPTER 4. ELECTROSTATIC FIELDS IN MATTER Problem 4.29 : (2) Eq. 45.3 Fa = (p2- WE: Fe P By, 8108 2 B= 5 b= EP oa. There 3p Frey” tele (=)I To calculate F;, put ps at the origin, pointing in the x direction; then pi is at -r2, and it points in the -9 direction. So F, = (p)'V)E2 i 3 we need Ey as a function of 2, y, and 2 a area | From Eg. 3104 By = pay. -ef. where F = 2849 +2%, p, = —pry, and hence = aneg (see = Pa [~3ayR + (2? — 2y? +29) F- Sy2t Gry eer eye ray 2yl-Boy + (2? —2y" + 24)9 — ayed] + 2-828 —Ay9 aay} 7 (2%) - a] ‘These results are consistent with Newton's third law: Fy = —Fy (b) From page 165, Nz = (pz x Ei) + (r x F2). The first term was calculated in Prob. 4.5; the second we get from (a), using r = rJ: Pe eg, ex Fe crg)x (2 px By = PP ay rea = (9) x (PE 4) = 22, © Ancor? ‘his is equal and opposite to the torave on ps due tops wth respect othe center ofp (ee Prob. 4) Froblom 430 " te SHEE No at hed ne tuto ep howe be Bis perpendicular to the surface of each conductor. 1 , po Problem 4.31 Pekr=k(zk+y¥+z%) => py ‘Total volume bound charge: [Qyo1 y= Pa, At op surface, 2 2 Ces [op = ofa] on all ix utes VP = -k +149) = F3E mF atx ke ee oe 7 on [ae ewe atta (r-4) oa 50] 1905 04 = Sa 2) Xe ust = 0444 R*) =|q—%*—.] The compensating negative charge is at the center: Quart = o0(4eR?) fs x | P ig negati BF Tex xe fp Xe dr=-B* [eear [om ie [Pe Thx Probiem 4.33 Ell is continuous (Eq. 4.29}; Da is continuous (Eq. 4.26, with oy = 0). So Be, = Bes, Dy, = Dyn > 6 By, = Ey, and hence En _@ lant ear ee ae (ris air and 2is dielectric, tan0s/ tan 4, = ¢9/¢ > 1, and the fcld lines bend away from the normal. Tis is the opposite of light rays, o 9 convex “lens” would defocus the fl es Problem 4.34 ~ In view of Eq. 4.39, the net dipole moment at the center is p! = p— pp = aie potential produced by p’ (at the center) and as (at R). Use separation of variables: p. We want the Outside: V(r0) = 5° PR (0080) (Ba. 3:72) 1_ pcos PgPOE + DV aur'ncose) (Bas 8.66,3.102) Inside: V(r,6) ae = AR, or B= RA, (#1) V continuous at R => wv) _ av or or 86 CHAPTER 4. ELECTROSTATIC FIELDS IN MATTER n+ fy ain xetARE! (11); or = (21+ DARI = xtARI! > Ay = 0 (C4 2). San paROTE” Reber A abe a= mbes 7 zee fi+2(esh) 2] 0, s0 Es = 0, so Vo — Va = constant. But at surface, Vo = Vi, so Va = Vi everywhere. ged Problem 4.36 -E) and Vs (Bo (@) Proposed potential: | V(r) = Vo%.| I so, then |= WV = VoAP| in which case ; R say =| 2x00 : in the region z <0. (P = 0 for z > 0, ofcourse} Then oy = eoxeVazes (a) =|— 2X. | vote: points out of dielectric + A= ~#.) This op is on the surface at r= R. The flat surface ¥=0 earties no bound charge, since A = @ 1 # Nor is there any volume bound charge (Eq. 4.39). If V is to have the required spherical symmetry, the net charge mast be uniform: och R? = Guat = Arey RVo (since Vo = Quos/4"¢oR), 80 ove = eoVo/R- Therefore or (coVo/R), on northern hemisphere } (coVo/R)(1 + Xe), 09 southern hemisphere = 9540, = co¥/Ris uniform (on the northern hemisphere os = 0, 0 ~ eoVo/Rs coxeVo/ R, 80 a) = eVo/R). The potential of a uniformly charged sphere is = Gio, _ seal eR? * Geer 4meqr (6) By construction, 0 ‘on the southern hemisphere 7 Ye (©) Since everything is consistent, and the boundary conditions (V = Vo at r = R, V0 at co) are met, Prob. 4.35 guarantees that this is the solution, 87 (2) Figure (b) works the same way, but Fig. (a) does not: on the flat surface, P is not perpendicular to, s0-we'd get bound charge on this surface, spoiling the symmetry. Problem 4.37 a inv, this Xe = 4. Since the sphere is tiny, this is essentially constant, and hence P| Bexe (Ex, Eat = 5s pt Ys ly and hi Taba Ex 47). r= | Btn) (a) (2a) (eBtp) Ga) ORL + wY)14 TFx/3 (es) aire Problem 4.38 The density of atoms is N= cghepr- The macroscopic field Eis Ear + Base, where Byer is the average field over the sphere due to the atom itself P=OB aie + P= NoBo. (Actually, it is the field at the center, not the average over the sphere, that belongs here, but the two are in fact equal, as we found in Prob, 3.414.) Now P Eset oR (4. 3.105), 80 ee « Nav - pig iirBaet Bae = (1- ware) Ee (0 ~ 3) Bae So Na P= Go Wajaaye = OX: and hence ep x © = (= Na/3ea) Solving for a: cole "= NTF xP is Problem 4.39 For an ideal gas, N = Avagadro's number/22.4 liters = (6.02 x 10%)/(22.4 x 10-8) = 2.7 x 105. Na/eo = (2.7 x 105)(4rre0 x 10-%)8/eo = 3.4 x 10°48, where f is the number listed in Table 4.1 H: 6 =0.667, (3.4 x 10-4)(0.67) =2.3 x 10-4, x, = 2.5 x 10-4 He: $= 0.205, (B4 x 10-4)(0.21) = 7.1 x 10-8, ye = 6.5 x 10-5 ment is quite goo . (84 x 10-4)(0.40) = 1.4 10-4) yp = 1.8 x 10-+ f ABEEeMENt Is quite good B=164, Nafeo=(34 x 10-1)(1.64) = 5.6 x 10-, ye = 5.2 10-4 Y Cc 88 CHAPTER 4. ELECTROSTATIC FIELDS IN MATTER Problem 4.40 : (@) w = £ peuenwAT du _ (kT)e~™/*T [-(u/KT) — UP Pree du ~eEe=weT PE [e-PE/AT PEAT) + [(pB/kT)e-PF/*T + (DE/RTY EDAT — @pETRT kr - peo (22) PENT 4 @-vE/AT ~98 (Soares i 5 aT (P-E)(B/F) —(uy(B/E); E/T. Then y = cothz—1/z. Asz +0, 0, s0 the graph starts at the origin, with an initial slope of 1/3. As x -» 00, y -> coth(a0) = 1, so the graph ‘goes asymptotically to y= 1 (see Figure) a pe/kT (b) For small 2, = $2.50 5 © Bo P= SEB = coxeB => Ps proportional to B, and| x. = For water at 20° = 293K, p = 6.1 x 10-°°Cm; N = molecules = molecules ¢ moles Bram | N= (6.1 x 10%) x (§) x (10°) = 0.33 x 10; xe = yp SSSR ess = [12.] Table 4.2 gives an experimental value of 79,50 its pretty far of For water vapor at 100° = 373K, treated as an idea! gas, "me (2.11 x 10%5)(6.1 x 10-9)? (22.4 x 107) x (388 BB) = 285x107? m ‘Table 4.2 gives 5.9 x 10-*, so this time the agreement is quite good. Chapter 5 Magnetostatics Problem 5.1 Since v x B points upward, and that is also the direction of the force, q must be [positive,] To find R, in terms of a and d, use the pythagorean theorem: (Rd +e =e Rome eta a R—Lte R The cyclotron formula then gives in - a| opie +) p= qBR=|qB Problem 5.2 ‘The general solution is (Eq. 5.6): ult) = Cy coset) + Casin(ut) + Ze+ Cy; z(t) = Cacoslot) ~ C1 sin(ut) + C4 (a) v(0) = =(0) = 0; 9(0) = B/B; 2(0) =0. Use these to determine C1, C2, Cs, and Ce (0) =0 = O, +03 =0; §(0) = uC, + E/B = E/B = Cz =0; 2(0) =0> C2 + C= 03 =0; 30) = 0, and hence also Cs = 0. So[y(t) = Bt/B; z(¢) = 0.] Does this make sense? The magnetic force is q(v x B) = ~q(E/B)B2 = ~gE, which exacily cancels the electric force; since there is no net force, the particle moves in a straight line at constant speed. 7 (b) Assuming it starts from the origin, so Cy = ~Ci, Cy = —Cp, we have £(0) = 0 > C, = 0 > C; =0; aft) = se psin(ot) + Be, 2) =~ coslot) + 3, of | y(t) = a ut —sinfut]s z(t) = 5 east) | Let a = £/tus, Then y(t) = Blut ~ sin(ut)};_2()= BIL ~ coswt)]; w= Rat) = Szeto | BVH ~Becostut) -> (y — 2But)? + (z — 8)? = 8". This is a circle of radius # whose center moves to the right at constant speed: yo = 2Bwt; 2 = 8. » Cy = 2Q= (9 0) = 0) = 5 + Cw = 5 89 90 CHAPTER 5. MAGNETOSTATICS & sintwr) (1 +t ~ cos(ut)] 2(8) = = = RZ Fesintwo.|ut0) 0B; thea {y ~ B(1-+.t)) = ~Beos(wt), 2 = Dsinlwt): fy BU Fu + Let 6 (6. This is a circle of radius # whose center is at yo = B(L-+t), 29 = 0. ; : le | 8 | y y ) re © Problem 5.3 ~ (0) From Bq, 62, F = qIB-+(v xB) =0- B= 0B s|u= 2 (b) From Eq. 5.3, mv = ¢BR=> Problem 5.4 Suppose I flows counterclockwise (if not, change the sign of the answer). The force on the left side (toward the left) cancels the force on the right side (toward, the right); the force on the top is JaB = Iak(a/2) Tka? /2, (pointing upward), and the force on the bottom is IaB = —Tka*/2 (also upward). So the net force is F =[Thot, Problem 5.5 because the length-perpendicular-to-flow is the circumference. a [ feteas= tna [ds tron > 0 53 = ersind} [J = pursind d,| where p = Q/(4/3)xR® Problem 5.7 dp dt says V+ (23 2 pete f (Se) rere ~ few str tr he otis ent. Now rete #5 2(V-J)4+J- (We). But Vz =%, so V- (2) = 2(V-J) + Je. Thus fy(V -Jjzdr = [ V-(eI)ar— f Jedr. The first term is f,2J -da (by the divergence theorem), and since J is entirely inside V, i is vero on the surface S. Therefore f,(V -S)zdr = ~ fy Je dr, of, combining this with the y and = Sy Sdr. Or referring back to the fist tine, P= 3dr. qed = components, fy(¥ Med Problem 5.8 v2u01 aR (a) Use Bq. 5.35, with 2 = R,0, = -0, = 45°, and four sides: B = (b) 2=R, = ©, and n sides: B= a1 (©) For small 8, sind 8, $0 as n +00, -» TOE (E) o| MOL) (ame as Ba. 5:38, with 2 = 0). Problem 5.9 (@) The straight segments produce no field at P; The two quarter-circles give B =| #0! (#G- 121) | coup). () The two half-lines are the same as one infinite line: #24; the half-circle contributes 22 aR a T(,,2 SoB= et (i+2) {isto the page). Problem 5.10 (a) The forces on the two sides cancel. tthe bottom, (up). At the top, B = Fa ale 5 (down). The net force is Ir(s +a) (b) The force on the bottom is the same as before, jig Fay (29 + dy 8). Bus the # component cancels the GF = I(dl x B) = Ide + dy9 +24) x (jat2) Pa 2 fsleep2} mb fide. Hore y = VBe,s0 hy pol? (siv3 + a/2 Wan ava eo the rangle | #2 = Problem 8.11 ‘Use Ba, $38 for ang of width ds, with I+ nde For an infinite solenoid, 62 = 0, @1 = x, 80 (cos #2 ~ cos6;) = 1 - (1) = r 92 CHAPTER 5. MAGNETOSTATICS Problem 5.12 Magnetic attraction per unit length (Eqs. 5.37 and 5.13): fy 1 Electric field of one wire (Bq, 2.9): E = =+-4. ectric repulsion per unit length on the other wire Lx 1_ 2. they balance when pov? racg ag They balance when 1 Problem 5.13 (a) fB-a= Bars Qrks? st ‘Qna® 37 Flore 0, $0] B Problem 5.14 By the right-hand-rule, the field points in the ~$ direction for z > 0, and in the + direction for z < 6. At z= 0,B =0. Use the amperian loop shown: pola, =poJay, for z > +a; _ amperian loop alee { tyoJa¥, for 2 > -a. } a z ty Problem 5.15 — + s The field inside a solenoid is yonZ, and outside it is zero. The outer solenoid’s field points to the fet ( whereas the inner one points to the right (+2). So: () [B= wof( — na) 2,] i) [B= —notne 4} i) [B =O Problem 5.16 From Ex. 5.8, the top plate produces a field joK/2 (aiming out of the page, for points above it, and into the page, for points below). The bottom plate produces a field jwoK/2 (aiming into the page, for points above it, and out of the page, for points below), Above and below both plates the two fields cancel; between the plates they add up to woK, pointing in. (a) [B = poov (in)] betweem the plates, [B= 0] elsewhere. (b) The Lorentz force law says F = [(K x B)da, so the force per unit area is f = K x B, Here K = ov, to the right, and B (the field of the lower plate) is jo7v/2, into the page. So [ fm = 100?v*/2 (up)- 93 (©) The electric fel of the lower plate is o/2co; the eeciic force per wnit area on the upper plate is Tina They balance if poy? = 1/eq, ot [v= 1/ ais =e] (the spend of light), asin Prob 5.12, Problem 5.17 ‘We might as well orient the axes so the field point r lies on the y axis: ¢ = (0,y,0). Consider a source point at (2',y's2") on loop #1 2R+y—V) G2 dl = del R4 dy! 9; = (-2' dy’) + (2' dz") ¥ + lly — yz! +2 dy')2. ! dy!) + (2! ds) 9 + Uy - de! + dy (@PFo-vP ee Now consider the symmetrically placed source élement on 4 loop #2, at (2',y',—2"). Since 2/ changes sign, while every- thing else is the same, the % and ¥ components from. dE and dB, cancel, leaving only a % component. ged With this, Ampére’s law yields immediately: fa [ines inside ase Be 10, outside (the same as for a circular salencid—Ex. 5.9). For the toroid, N/2rs = n (the number of turns per unit, length), so Bq, 5.58 yields B = pon! inside, and zero outside, consistent with the solenoid. (Note: N/2n3 = n applies only ‘he torcid is large in circumference, 50 that ¢ is essentially ‘constant over the cross-section.) Problem 5.18 Ik doesn’t matter | According to Theorem 2, in Sect. 1.6.2, [ J-dais independent of surface, for any given boundary Tine, provided that J is divergencoless, which itis, for steady currents (Eq, 5.31) Problem 5.10 (5) wbere (a) p = 2282 _ charge atoms, moles Volume ~ “atom "mole ” gram © = chargeof electron = 1.6 x10"! N= Avogadro's number 6.0 x 10° mole, M > = atomic mass of copper = G4gm/mole, d= density of copper. = = S.0gmfem? tenaneai (G 2) = [Ld Ce iweha] =tepaen 1x 10-¥em)s,} or about 39 cm/bx. This 0) d= sip = 95 = 5 = aaa eipTgTA TON ~ [PLO em/al or bout 99 emp is astonishingly small—iverally slower than a sails pace. : “) (0) From Bq. 5.37, fra = a \ =e 4 CHAPTER 5. MAGNETOSTATICS [= 3.00 108m, Hee = & = (20° 108)" fia] é vw fe = (11 x 10)(2 x 1077) 2x 10° Nem. Problem 5.20 Ampére’s law says V x B= od. ‘Together with the continuity equation (5.29) this gives V (Wx B) = poW -J = ~yoDe/t, which is inconsistent with div(curl)=0 unless p is constant (magnetostaties). The other Maxwell equations are OK: V x E = 0 V-(V x) =0 (V), are as for the two divergence equations, there js no relevant vanishing second derivative (the other one is curl(grad), which doesn’t involve the divergence). Problem 5.21 ‘At this stage I'd expect no changes in Gauss's law or Ampére’s law. ‘The divergence of B would take the form [VB = copm,] where py, is the density of magnetic charge, and ag is some constant (analogous to ¢ and ji). The curl of B becomes ['V x E = Boras] where Jn is the magnetic current density (representing the flow of maguetic charge), and J is another constant. Presumably magnetic charge is conserved, $0 fm and Jm satisfy a continuity equation: V -Jm =—Opm/@t- ‘As for the Lorentz force law, one might guess something of the form gn(B + (v x B)] (where gn is the magnetic charge). But this is dimensionally impossible, since E has the same units as vB. Evidently we need to divide (v x E) by something with the dimensions of velocity-squared. ‘The natural candidate is 2 = Iequo: |F = 4e(B + (v x B)] tam fp (v x »| In this form the magnetic analog to Coulomb's law reads F mF, 50 to determine ay we would frst introduce (arbitrarily) a unit of magnetic charge, then measure the force between unit charges at a given separation, [For further details, and an explanation of the minus sign in the force law, see Prob. 7.35.) Problem 6.22 a8 [pen St aes = * a 1, [zt Vea A ino ver gape Co B= oA Hol a a aoa an aT TE IITA nt Varee fare * uals [ 2-J@PRR 1 -V@yrF 1 a [GPP ee Jars Tar sare :) a (as in Eq, 5.35), 95 (7 Problem 5.28 aBavxa=t Zina ae -nba(D] 4 Fein oa Deke (eB) = HB. (V x8) -2-(7 xB] = 0, sae WB = 0 (B union) and V xr = 0 (Prob. 1.62). Vx A = ~5V x (rx B) = (B-V)r -(F- V)B+4(V -B)—B(V-r)). But = = 0 (since B is uniform), and V-r = OF 4 5Y 4, 9 (e-V)B = 0 and VB = 0 (since B is uniform), and Yr = FE 4H 4 Be (wiv yr= (262 + By + Beg.) eRv4s 428) = BHR, IHBA=D, SOKA ged Problem 5.25 (8) A poinsin the same direction a6, andi function only of» (the dtance from the wie). In xine A(s)2, 30 B= VX A= -%46 = fot 4 (te field of an infinite wire). Therefore 1+1+1=3. Finally, 1 #(8-38) = coordinates, then, A. Be ap, and A(e) = ~422 In(s/a)2| (the constant a is arbitrary; you could use 1, but then the units A, OA: 5 ; aso vx = Mol look fishy). VA = SE =O. 0 Fe b= O=BY (0) Mere Anpéce’s law gives f B-dl= B2x9 = pale = pol 18? = potas? = Male Sea ase Sean onan re pee) eae eee = ~~ Ha. Hore bis again arbitrary, except that since patina Bowls a= eee — 0), Here b is again arbitrary, except that since A. (- nus be continuous at, Mol in(R/a) —#%), which means that we must pick a and b such that 2 aera ol (2 py : (O/B). Vil use a = aR Me Brose 2in(R/>) =R.Yhen| A = ~wLinis/a)i, for 82 R. Problem 5.26 K = K+ B= £45" ¥ (plus for 2 <0, minus for z > 0). 2 A is parallel to K, and depends only on 2, so A= A(z) X osy, ajas | 245 B=VxA=| afar ajay a/az |= S49 a Az) 0 0 v 0K | wil do the job—or this plus any constant K Problem 5.27 fa) v- ts [-v-(2) ae. 9: (2) = ev-n43-v (2), wie ae abe : -#e()=-e() {sa function of the source coordinates, not the field coordinates. And since + = 96 CHAPTER 5, MAGNETOSTATICS ©-(2) =v" (2). nv: (2) = Heo'3y43-0 (2), and 6a = in manta 88,8 v-(2) ==07(2)sand tance dorm son VA = =H fo (2) a= fa ovxasit [vx (2) eek [[xn wae po f3xa,, a function of x), ana v (2) =-3 (Pa tion, sov x a= 2 [Pan = i °()| dr’, But V x J =0 (since J is not Lv to vat [ve (2) a uv" (2) =20°(") tome agai. D i conan at a i ation with respect to r is concerned), and V? -4n6°(a) (Eq. 1.102). So VA, 2 f 30) [-ar5%t)] Sail Problem 5.28 . pol = JB - VU «dl = —[U(b) - U(a)} (by the gradient theorem), 50 U(b) # Ula). qed Foran infinite straight wire, B= Bed ‘would do the job, in the sense that wy = Mal gig) = wet 3 = B. But when ¢ advances by 2x, this function does not return to its initial value; it works (sy) for 0'< 9 <2r, but at Orit “jumps” back to zo Problem 5.20 ‘Use Eq. 5.67, with RF and o + pdF: ait 5 aay 4 ena 3 [eae = (282) sno (2) +505] = = Howe {_} + pee ate £2) 6} B= vx Mage {TS [sno ono (4 =) e-3 [rae =)|@ RE rea = soun|( 2) cots (© -22) an] 20t 9= Poa - (EE e=n- (Ce) Problem 8.30 (a) me Fy Welzns2) = — JE Fya'sy.2)da! + Cily,2). ea hy Wylay2) = + 6 Plesysdde! + Ox(y2) ‘These satisfy (ii) and (Gis), for any C; and Cit remains to choose these functions so as to satisfy (i): 97 (z', y, 2) OC, OF.(z'.y.2) 1 ey 2 8 a SEI 4 9G. f° RELI a bw ley, bat BE 2B 4 PE = “OF! ed 4 On aC, OF, (2% 2) f RU ae fae + FFE = Plea soe f' HEM?) del = Fy(e,y52) ~ Fe(0,4s2), 80 1 C2 1 ’ at 9B FE F(0,y,2). We may as wel pick C= 0, C(gn2) =f” Fa(O.',2) dy and we're doe, with W.=0; Wy [Fletanvae' Ww. [[ s000.0a/- * yla'sy,2) da! OW, Wy), (OW. wvxwe (Fe -Sa)e+ (Te or y = [atone f Awe - [gee wlerpes vassal] + UFelan2) 012 Bas 92 a0 the term's [one + f° SHEE al = Oy) + lee) ~ Fada 2 wVKWahy * OFyla v2) ay f PG!) ae! £0, * OFs(0'.9.2) yr, [1 DE a(O¥'2) Cues +f be oz Ode Ajay fz | =yX4zH+re=F.V ("0 2/2 @/2~z) | ( Probiem 5.31 (2) At the surface of the solenoid, Basove = Evidently Bq. 5.74 bolds. (b) In Bq. 5.67, both expressions reduce to (yp Rwa/3)sinO@ at the surface, so Eq. 5.75 is satisfied. oa okie (_ int) Al. toRwe Fhe 2 oS) 4, sind, I = Si“ sindg. So the left side of ale 3 Eq. 5.76 is ~ypRue sind. Meanvihile K = ov = o(w xr) = owRsin0d, so the tight side of Ba. 5.76 is =pgauRcin 6, and the equation is satisfied Problem 5.32 ~ Because Aasove = Absiow a evry pint of the sutfce, it follows that $4 and SA are the same above Oz KG. oKxh MoK & fh 0, Beetow = pon! and below; any discontinuity is confined to the normal derivative aA, Batove ~ Bostow Wok (-§), 80 Apes = 244 es —2Aszue = —yyK. Thus the normal derivative ofthe eom- ponent of A parallel to K suffers a discontinuity ~poK, oF, more compactly: . cae vst | Problem 5.83 (Same iden as Pro. 3.33) Write m = (m-#)# + (an-8)8 = meoed ~ msindO (Fig, 5.54). ‘Then ‘lm 7) # — m = 3m cosB ~ mos? +msindG = Imcosdé + msin #6, and Eq. 5.87 Eq. 5.86. ged 98 CHAPTER 5. MAGNETOSTATICS Problem 5.34 a (a) m= Ja =[InR?z. bo In? ane (0) Be (2c0s68-+ sina d) (©) On the z axis, 0=0,r = 2% (for 2 <0, 0 =, # = ~2, so the field 2 (for z > 9}, s0| B= is the same, with J? in place of 2°). ‘The exact answer (Bi 8.8) Teduces (for = 3 R) to B ~ pol R?/2e), so they agree Problem 5.35 For a ring, m= Inr®, Here I + ovdr = owrdr, so m= fi" xr2our dr =[rowR*/4, Problem 5.36 ‘The total charge on the shaded ring is dg = o(2n Resin #)R db. ‘The time for one revolution is dt = 2n/ws. So the current, inthe vagis = A= oof? anda. Te eno the tng is m(Rsin8)®, so the magnetic moment of the ring is dm (owR? sin 6 d8)x R? sin? 9, and the total dipole moment of the a shell is a eo ‘a ms owe ff sin? 649 = (4/3)ounRt, or|m = Zouk ‘The dipole term in the multipole expansion for ATs There- pS wodn ogsind = pawh! sind 5 fore Aap = $2 owns Sn! S82 64, which is also the ezact potential (Eq, 5.67); evidently a spinning sphere produces a perfect dipole field, with no higher multipole con- Problem 5.37 Fhe field of one side is given by Eq. 5.35, with » + JF GTAP and sin By = ~ sind = Perea patel ‘To pick off the vertical tr ESTO component, multiply by sing = cee for all four sides, multiply by 4: |B For > w, Bx MEG. The field of a dipole [m = Tu? for points on the 2 axis (Eq. 5.86, with r -+ 2,8 2,0=0) is wmey ane Problem 5.38 ‘The mobile charges do pull in toward the axis, but the resulting concentration of (negative) charge sets up an electric field that repels away further accumulation. Equilibrium is reached when the electric repulsion on a mobile charge q balances the magnetic attraction: F = g{E + (v x B)] = 0 + E = —(v xB). Say the current (~~ isin the 2 direction: J = pv (where p~ and v are both negative) JB dl= Bans = poJnst = B = HOP - da = B2nsl = +(p, + p_)xs'l + B= Feet ess. 0 1 © a eee -@) —7 ps. In this naive model, the mobile negative charges fll 1 ‘ Beg Pt + P-)88= Evidently py = =p. (1 smaller inner cylinder, leaving a shell of positive (stationary) charge at the outside. But since v < ¢, the effect is extremely small Problem 5.39 (a) If positive charges flow to the right, they are deflected [down, Jand the bottom plate acquires a positive charge. (b) quB = gE + B= 0B = V = Et =[vBt,] with the bottom at higher potential. (c) If negative charges flow to the left, they are also deflected down, and the bottom plate acquires a negative 7~ Siaige,_The potential diference is till the same, but this time the top plate is atthe higher potential. Problem 5.40 From Eq. 5.17, F = I {(dl x B). But B is constant, in this case, so it comes outside the integral: F = 1 (fal) x B, and {dl = w, the vector displacement from the point at which the wire first enters the field to the point where it leaves. Since w and B are perpendicular, F = /Bw, and F is perpendicular to w. Problem 5.41 ‘The angular momentum acquired by the particle as it moves out from the center to the edge is us [$a = [nam [ooxmat=frxaexma=ofextaxm=a[fe-ma- faa] But r is perpendicular to B, so r-B = 0, and r-dl = r-dr = ) = fd(r®) = rede = (1/28)(2erdr). f SoL= -£f Baar dr = -£ [aa It follows that where & = f Bda is the total flux. Im particular, if ® = 0, then L = 0, and the charge emerges With Zero angular momentum, which means itis going along a radial line. ged Problem 5.42 From Eq. 6.24, F = [(K x Baye) da. Here K = ov, v = wRsinO 6, da = R? sin d0 dé, and Bave = }(Bin + Boyt). From Eq. 5.68, 100 CHAPTER 5. MAGNETOSTATICS 2 ne For Rue = Zico s(cos* — sind), From Eq. 5.67, = uA ay x (Hoioo sind 3) _ uoRwo [_1_ 8 (sino Ban = Wade vn (MEME) = ee aie (“) 2 Hole (rcoso¢-+sin06 = HOR (2 cos08 + sin 6) (since r = R) oko Bae = (4cos0# ~ sin 4) voltae 6 a Kx Bye = (ulna ( ) [a> e4eosee —sin9d)} = BY cos8 + sind) sina Picking out the 2 component of 6 (namely, ~sin8) and of F (namely, cos@), we have (Kx Bue): = — 22 (eu)? sin® 8.c088, 80 a2 F, 20 ory R? f sin? 8s dad = — Wok? 2 (2) Brobiem 5.48 (@) F=ma=q(v xB) = 10 dlr cy 99, [ya BO aa mr (v x2). (Beene a yaw <0, Bat av = 18 = EZ aed aQ Hodedm dr) _ HOGG Hodedm (VY dr (6) SP = rr xv) tame xa) ~ Hatem S (E) = 04 Has x — Moet (EES) = Het { Fay — (e-vyq]~ E “Abs +P) nov @) @Q-6=Qe- rertand v= 4 =Fe-4r60-+rsind6d (where dots denote diferentation with respect to time), 50 so (rx ¥) 6 =0. But (-1? sin ad) O + (776) 6. "Therefore (r x v) aed (i) Q-#=Q@-#) = mF xv) a — Heels 8), But &-# = cos8, and (rx v) Lr + (rx v)-# = 0, 80 cosa = Hall, or Q-= ~HOFEI. And since # is constant, so too is @. aed (ii) Q-6 = Q(8-8) = (rx) -6 EH (F.6), But -B = sind, #4 = 0, and (¢x¥)-0 = 1? sin8d Q Saat k (from (i), 89 ~Qsin® = ~mr* sind > = 25 = 5, with| ks S = — Belen ta P84 eit d= 0and 5= 5, Paint pet ye Rsin2e ~sinteS au - SE 101 “gt <0 [Ey se: 0 ean [00> 6- t= Hise ean tan 0 7) = See Le iaiM anne Problem 5.44 + see{(Y ~ do) sing] = sind Put the field point on the x axis, sor = (s,0,0). Then p= [ED an do = Rava K = KO = K(-sin 9X + cos 9):4 = (s — Reosd) k — Rsin gy ~ 23. Kee = K| -me cos 0 | (s— eos @) (-Rsing) (-2) K[(-z008g) + (~zsing)¥ + (R— sc084)2]3 = 244 Ri+s-2Rscosd. The:x and y components integrate to zero (z integrand is odd, as in Prob. 5.17) (R—scos) oe eke | pps Bscosaer Oe . a 7 m= cn osono{ [arta } Cc where = RP + —2Mee0sd. Now | wea eral a 1K R (R= scosd) Re I, (R45 Bhs cosd) = We a ees is wo 4 ages [YH F tanteo/2) ||" fp a+beosd lo atbcosd fa? — 5? a+b Jar sot] di, (R~ 50s) = 3h [(R? — 58) + (+ 9? ~ 2Rs 0s8)] on Tate b= -2Rs, soa? — 0 = P+ DRA? + ot — ARE = RY ORE 4 a = (RY 9, VPP = [BP ok [(R2 ~ 5%) woK (RE? 2, = Hes e=s ze +24] aK ( +1). Herea = R48, aa [ a+b Inside the solenoid, 8 < R90 B, = “ME (141) = pK Outside the solenoids > R50 B, = 49K (-141)=0. Here K = nl, so[B = jgnTa{inside), od O(outside) | (as we found more easily using Ampére's law, in Ex. 5.9), 102 CHAPTER 5. MAGNETOSTATICS Problem 5.45 @ the source point be = Rosé — Rsingy, and the field point be r = ReosO%+ Rsind¥; then'a = Ri{(cos0 — cos 4) & + (sind + sing) ¥] and dl = Rsin gdp + Reosbdd§ = Rd9lsin $& + cos). Ch\ 2 VV x y dixa = Rdg} sing cose 0 KX A, (cos ~ cos) (sin@-+sing) @ ¥ = RX(singsind +sin® 4 — cos# cos + cos? 4) dbz, = FE (1 +sinB sin d — cos0 cos 4) dba = R? (1 — cox(9 + 4)} dd. B= bal = tale waif {1 = cos(0+4)) iol 2 fC ar bee aeees oF * &QR PP ree =) ele a 6+¢\] ol ,, [tan (2°)] 5 ~ ik f Faeronl~ Ten 4 {2 fan ( 7 2 er [a Problem 8.46 (a) From Eq. 5.38, laevahesar ies 3) 20 _ wl oz 2 [2 + (a/2+ 2°? 5/2) 2(d/2 + 2) -3/2)2(d/2 ~ 2M(— [Re + (aj2~ 2)? — soll {jaz}, (a2) 2 les (aja ay? © [e+ 2-9? OB _ Buf R? -dj2 / 7 aE lo 2 | mes aapr? * ies ara (b) Differentiating again: PB _ Seal =(a/2+ 2)(—5/2)2(d/2+ 2) ad 2 Uns (a2 + ey? [F? + (a/2+ 2)?" + =1__, (a= 2-322 e+ Gp] (Re + eB = Sao R? ~2 2(6/2)2(d/2)°2 | ___ 30 R? (- +) OF lico 2 re ara? * ire + (aay? J ree apy"? oo = — uth? ). Leroit in which ease aay OO) =a} pe ee ee BO) = Ay facta dar} Wola 103 Problem 5.47 (a) The total charge on the shaded ring is dg = o(2nr) dr. The time for one revolution is dt = 2x /w. So the current in the ring ist Morar, Pom By 5.28, the magnet fl fis Ae zi ting (poo 8) B= Mr a ry > and the total field of the disk is woow fdr B= Tb Geer , sodu=2rdr. Then woow fP udu ai, uray {b) Slice the sphere into slabs of thickness t, and use (a). Here o = pt = pRsinod;R > Rind, z + 2 - Reosd. First . ‘hein rewrite the term in square brackets: a (#4222) -2] ass) Vitee |)" vate Vibe gl ° Rp =|VBee - VEv?- ae But R24 24 Rsin® 84 (2? ~ 2Recos8 + RP cos? 8) = RP + oo 2 2Rzcos8. So soph, [* S sinda | (P42? — DRe cod = (e = Reos?) phe, | { +i BRecond - ISTO -(e ~ 2cos0) Let u = c0s0, so du =—sin8dd; @:0-+ xu: 1-1; sin®@ , (R2/2)(1~ w?) Vie eee - Pw) du voor [| * Vie th** = popRw [a-Sa@ -hi-h +4] t= [RRR ae = 5h (RP 4 2? ~ aRewy" | eee eee L 2 = ~ aa [(F 42? 2R2)? — (R24 2! + 2R2)"") = ~ 5 [le - R) = (2+ RY) = wg (2 = 92 Sane ~~ Bal RY) = Leas? + PD) ho= =o VFR! =-Ele-B- e+ =2 fa CHAPTER 5. MAGNETOSTATICS Cc a= | aeescoe appar (UE + 2) 44 + 2 RRew 4 S2R=)eA] VA Tea = nappa {BR +2) + 8ree(r? +2") + 2A] (eR) ~ (BIR? + 8)? — 8Ra(R +24) + BRA (e+ AD} 2) — to(n? + 24)? + 24R82}} «_3pa,2 3002) 4 5 pazz_ pt). 4. (pry 5,2 aa 2 yam (HP) = ga (ge) eee fl oe [21,2 , py A272, RE 4 (oy 52 Be = waliow | 502? +R) ~ Fe Fags (#52) -%| oR? RP Rt RR ~ votes (se 322 ipa * i) aR’ __@ = pope. Bat o= Gay so] B Problem 5.48 B= Hl (MHA, Rosh + (y— Raing)¥ +22. (For simplicity Pl drop the prime on $) a 2 = RB cos? 6 + y? — 2Rysind + RBsin? d+ 2? = RF + y? +22 ~ 2Rysing. The source coordinates (c',y', 2") Os d= 0. Sod’ satisfy 2° = Reosd = da! = —Rsingd6: y’ =Rsingdbx + Roosddég Rind = dy! = Reos § de; y 4 Maa] -Rsinddd Reospdp 0 |= (Re cosddd)& + (asing dA)¥ + (—Rysingde +R d9)2 Roos (y~Rsing) py = HolRE ia cosp dé Mod Ra 1 1 Pg OE Jo Pas —aRysmy? ke Ry (Rye Ryan since sin @ = O at both limits. The y and 2 components are elliptic integrals, and cannot be expressed in terms of elementary functions. wolf sing do (R~ysing) dp an (RP 4? +2? ~ 2Rysing)” By ar 2Rysing? Problem 5.49 A From the Biot-Savart law, the field of loop #1 is B= seh j SAX® the force on loop #2 is Fe fs fay xB = Bang fe 2x (th x4 Now dy x (dh, x 4) = dl, (Aly 4) —A(dly “dl, 80 son (fffea-ao-fa.si ‘The first term is what we want. It remains to show that the second term is zero: a= (21) 84+ (Ya — 1H) F + (22 ~ a4) 8, 80 Val1/*) = 5 [lar — 24)? + (va — a8)” + (22 24)"] Hii g ns (2-21)? + 2 - 11)? + (@- 2)" _(=n) Pi lore sem enon @-a), + 3 * e 2in Sect. 13.3). ged 7 Problem 5.50 Poisson’s equation (Bq. 2.24) says V8V = —1p, For dielectrics (with no free charge), ps = -V-P (Bq, 4.12), and the resulting potential is V(e) = so the analogy is P+ eo, and hence V(r) = ed {There are many other ways to obtain this result, For example, using Eq. 1.100: v- G ) =-v (3) = 46a) = 4n6%(r —'), 2 Jv (8) « (Eq. 1.59). But V'V(w’) = —B(e!), and the surface integral > 6 at co, 0 V(x) before, You can also check the result, by computing its gradient —but it's not easy] Problem 5.51 (a) For uniform B, [5(B x dl) = B x [Sdl= (o) B= 216,00 fx a= (jeke- Gaaeeutaad Hr xB). (@) B= ol 5, Bar) = pee A = Hee eae 2 fA -tpvorjar-2 f vind a -Ba)we han = Hele x 3). But F here is th Zot = mat ® jut r here is the vector from the origin-~in cylindrical coordinates r = 88+ 28, So A = ~S2 [y(6 x 6) 42(@ d)}, and 7 5 T : (6x 6) =2, @« 6) =-8 So AaB (es— st ‘The examples in (c) and (d) happen to be divergenceless, but this is not the case in general. For (letting L = ff AB(Ar) AX, for short) V-A = -V- (ex L) = -[L-(V xr) -- (Vx L)) = r-(V XL), and V XL = £MV x Bln} Ad = [1 AV x B(Ar)} dA = no fe A258) dA, so VA = por fi MI(Ar) dX, and it vanishes in regions where J = 0 (which is why the examples in (c) and (d) were'divergenceless). To construct an explicit counterexample, we need the field at a point where J 7 0—say, inside a wire with uniform current. 106 CHAPTER 5. MAGNETOSTATICS Hore Ampér’s law gives Bre = pala = tades? => B= "9s, s0 ; ip 1048; ay oie =: = BE as ~ 9) Hod (1 a eee vA = E [sae =*3 FO. Concason: [does pot atonal yield WA = 01] Problem 6.52 (a) Exploit the analogy with the electrical case: Bo Ge RIMw-sF—B) (9.8409) =-WV, with y= ZOPSE (he. 3200) B = 4 i3(m.s#—m] (84.587) = ~VU, (Ea. 5.65) Evidently the prescription is p/eo— yam : |U(r) @ = (b) Comparing Eqs. 5.67 and 5.85, the dipole moment of the shell is m = (4r/3)woR*2 (which we also got awd RE £096, ti Hfor ¢ > Re in Prob, 5.36), Using the result of (a), then, |U(6) wo Ule) = ~ZuyowRe + constant, We may Inside the shell, the field is uniform (Eq, 538): B= 2u000R2, s Foowhr cos6]| for r < R. as well pick the constant to be zero, so | U(r) = [Notice that U(r) is not continuous at the surface (r = R): Uia(R) JiorwR?cos8. As I warned you on p. 236: if you insist on using magn places where there is current!) ) BuoowR® cos # Vou(R) = scalar potentials, keep away from 0 Ulr,8.4) =UG,9). 0 ip 18 = (a2) (1- sie) sn0 a= & ~ (Ez) C (0228) (1- $2) esr 0) =~ (22) (1 Zp) oe ot, (F258)? coo + 00 ) (= $e) reoso 109. ) (- Fz) +00 Sa) ome UG.) = (0). 107 rf But there is no way to write r? cos@ as the sum of a function of @ and a function of r, so we're stuck. The reason is that you can't have a salar magnetic potential In ategon where the current ts nonzero. Problem 5.53, === @y 0,7 xB =/od, and V-A=0,VKA=B 5A B [tar 2 1/B . and VW = 0 (we'll choose itso), Vx W= A 3]w= 2 f Bart (0) W will be proportional to B and to two factors of r (since differentiating fice must recover B), so I'll try something of the form W = ar(r-B) + r®B, and see if I can pick the constants a and f in such a way that VW =O and Vx WA. V-W=al(r-BYV +8) +r-V(r-B)) +8 [°(V-B)+B-VO%)]. Vr Bet gett las V(r-B) =F x (V xB) +B x (V xx) + (F-V)B + (B- V)r; but B is constant, so all derivatives of B vanish, VA=0,VKA or , dy , dz and V x5 = 0 (Prob, 1.62), 20 B= Or (nen on) ekarsen RAL naw vers (sored vad) ayy 2) = tex enys e2eb= te So V-W =a [3(e-B) + (F-B))+ 810 +2(¢-B)] = 2%r- B)(2a +B), which is 2et0 if 2a +f =O. Vx W = al(r- BUY x2) Fx V(r BY +A [F4(0 x B) —Bx V(r")] = a@0~ (e x B)] + 8(0—2(B x 2) = -(rx B)a-26) = plex B) (Prob. 5.24). So we want a — 26 = 1/2. Evidently a ~2(-2a) = 50 = 1/2, or a= 1/10,0 = ~2a= 1/5. Conctson:[W = © fee. B)— 27), | But eis is eran not wnigve) () VxW=A= J(V xW)-da=fA-da. Fw 7 {A da. Invegrate around the amperian loop shown, taking W to point parallel to the axis, and choosing W = 0 on the “ive f(t) nas = 28 cing 570A) al(s <2). won TRL rf won RP a tty Oo) S Re ul Hon aH 4 Mon in(s/R)s (> 2). Problem 5.54 Apply the divergence theorem to the function [U x (VV x V)]}, noting (from the product rule) that V-[Ux (Vx V)) =(¥ x V)-(V x U) -U-[V x (V x V)}: [rox eniees [ray ox) Uw x (wap dre fIUx(@ xv) sa ‘As always, suppose we have two solutions, By (and Ay) and By (and Az). Define By = Bz ~ By (and ‘As = Az ~ Ai), $0 that V x Ay = By and V x By = V x By ~ V x Bz = oJ Tod = 0. Set U=V= As in the above identity: 108 CHAPTER 5. MAGNETOSTATICS [607 Aa) 0 © As) = Aa 00 x Aa} dr f(s) (Bs) — AsV x Ba) [eer 0), or else Bis - fils x (Vx As)] da = firs x By) da. But either A is specified (in which ease A; specified (in which case By = 0), at the surface. In either case $(As x Bs) -da = 0. So J (Bs)? dr = 0, and hence Br = Bz. _ ged Problem 585 \ From Ba, 5.86, Biot = Bo ~ 2? (2eos0¢-+sin86), There f= Bola-t) — 2 rcs = (By — 2) cos \ fore Bs = Bole -#) ~ EP 2c0s0 = (8 Hore) cos 8. This is nero for all , when r = 2 given by Bo = SOL, or 7 Evidently no Sed lines cross this sphere. \ / ma etn Quetta Rien Mont = Masts (b) Because g is independent of R, the same re (or any other figure of revolution): |9 = 54. eh _ eh _ (1.60 x 10~29)(1.05 x 10-) a . — = [461 x 10-* Am? 2m2~ 4m 4(9.11 x 10° Bie aieals ()m= Problem 5.57 1 3 (a) Bue = wane [26 = game [0 * Mar agigfain = ptt ff fla} xm wigs [> {f Za} ar, noe tn: J doponds onthe source point r', not on the feld point r. ‘To do the surface integral, choose the (x, ,2) coordinates s0 that r! lies on the 2 axis (see diagram). Then 2 = \/R? + (2)® - DR cosd, while da = R* sin d9 dd #. By symmetry, the = and y components must integrate toner; since the 7 component off s 6086, we have 109 cos sind ref Viet ey tired fla = t/a : Ee Te lV AR ay aoe {nt + 2) + Re | VEE P= Bite — [RP +(e")! — Rel] VPP + ORs} — [aa 4| {[RE +(e) + Re] R= 2 - [RP +(e"? - Re (R42) (0) This ime > B80 Baw = —egSbeg 8 f (sx cis) dr = 2 1’). Thus Baye = Been _ged from the source point to the center (a = Problem 5.58 , (a Probie 851 gives the dpoe moment ofa shel: m= MEou' 2, Let 2 1,04 pdr and negate ar fh ae me Supt [edema -_@ But p= Grytpe: % o2m _ [na 200, 4n RE in SR coax eimang g [1m Quit ant g Ola 5 (d) Use Eq. 5.67, with R> F,o = mesa 4g _ How 3Q sing RS, 42 Gere sin® of" Oe sexe manis aaa aa (b) Bae = "and integrate: ‘This is identical to (c); evidently the field is pure dipole, for points outside the sphere tai2 (1-2) anos (1- fp) ned]. tho ores (@) According to Prob. 5.29, the field is B 10 CHAPTER 5, MAGNETOSTATICS obviously points in the z ditection, 90 take the z component of F (e036) and 6 (— sind) oe = HD ae as | (t= SB) oto (1-85) seo tins Buoy fT (( BRS ag, (RE of) ; 5 eat [ (§ ) costo 8 Sa) sin? sino ao mow ps "(16 294 sine SpowQ 1 f* 2 = Mae ne [ (Bos? + sin?) sine See a, LF (7 + 90080) sino pow BaP, (-1 cos 6 ~ 3oos*e) |” Problem 5.59 ‘The issue (and the integral) is idersical to the one in Prob. 3.42. The resolution (as before) is to regard Bq. 5.87 as correct outside an infinitesimal sphere centered at the dipole. Inside this sphere the field is a delta-function, Ad¥(r), with A selected so as to make the average field consistent with Prob. 5.57: . saydr= 3a = 02m, 4 Muon _ Boe are [AP Od ppd = fo ge A> Hy The added t2em is Problem 5.60 @ 1d a3dn@ [A= BS [eyrrscososer 7 dn 2 aT af (0) Amon = f& [tar = 22.2 (Prob. 5.7), where pis the total esse dipole moment. Tn magne tostatics, p is coastant, so dp/dt = 0, and hence Amon = 0. qed (©) m= a= H flr xdl) + m=} f(exd)ar. ged Probiem 5.01 For a dipole at the origin and a field point in the xz plane (p = 0), we have B= PA (eos + sind) = 12 i2cosd(sind x + cos02) + sin (cos 0X ~ sind 8)) = paps 8 cos & + (2e0s? @ — sin? 6) 4]. Here we have a stack of such dipoles, running from 2 = -L/2 to z = +L/2. Put the field point at s on the = axis, The % components cancel (because of symmetrical. ly placed dipoles above and below = = 0), leaving B MP (3cos? @ — tants [°° OSPF D ge, we Mi hele mo M4 |, F ment per Unit length: m = Inf = (ovh)nR? = owRa Rh => sk _ Sint cee a -scot@ dz = zm ‘Therefore = Fi = rowR®. Now sind 0 0 B = $2 (rowR)é [ “(3cos? 6 —1) dg = HOWE sf” (cos? @— 1) sin da $9 rou ne fr Los ) meow? 2s poowk® a oak? ‘2a 2 (— cos? 0 + c0s6) £080 (1 ~ 08? By) # = £0 By Si? Oy But sing 4 nd 08 ,90|B = ore hg V's? + (L/2)? als? + L/2P P72 J+ Ep Chapter 6 Magnetostatic Fields in Matter Problem 6.1 1 N =m) x By; Bi = 2255 [3(my -#)F — mu N= HOT g xa) = ET, Hetem, = 7a", mg = HT, So|N downward] (—2). Problem 6.2 dP = [dl x B; dN =r x dF = Ir x (dl x B). Now (Prob, 1.6): r x (dl x B) + dl x (Bx r) + BX (rx dl) = 0. But d{r x (r x B)] = dr x (r x B) +1 x (dr x B) (since B is constant), and dr = dl, s0 dix (BX 1) = rx (dl x B) — dir x (x BY}. Hence 2r x (dl x B) = d[r x (r x B)] - B x (r x dl) dN = 31 {alr x (rx BY) —B x (rx dl}. -. N= AT {fale x (r x B)) ~ B x f(r x dl)}. But the first cerm is zero (fd(-+-) = 0), and the second integral is 2a (Bq. 1.107). $0 N = -/(B x a)=mx B._ ged Problem 6.3 0 <- According to Ba. 6.2, F = (Bleut}t=m! and Beosd Ex (acim -#)6- 9) — m9), EG = sing, while my -F = myc0s®. -, Beosd = #2 53m, sind cas. F = 2x Rit 3m, sin 0086, Now sing = #, cos = VF=TB/r, 90 F But [Rx = ma, so F = 2mm, ~2ZEe, while for a dipole, R R). Boundary Conditions: te Win Ro) = Waye(Ri6), Gi) —BHem |, + ial, = Mi = Mb = Mood, (The continuity of W follows from the gradient theorem: W(b) - W( ithe wo points are ninesinally separated, this ast integral-+ 0.) { (= AR = fhe > B= RA, ti) Ba al gPe none) + Evarb Acoso Combining these: SW dle ~ [Hat Moasd. 1 = $0 A= and 34, = x Tersye noe Ar=0 (141), and 341 = Mx Ay = 2 Thus Win (8) = Mrcosd = he, and hence Hip = -WWin = B= poll 18 CHAPTER 6. MAGNETOSTATIC FIELDS IN MATTER r Problem 6.16 Hdl I;,. =1,s0H= ‘Total enclosed current, for an amperian loop between the cylinders: volt Xl gy 14 Kelosa = (14 xm) vo fBed= Halene = Holl + Xm)F => B = HOE Problem 6.17 From Eq. 6.20: fH -dl = H(2as) = Jy tigee (sa). ca (s a). (<0) pep = bs3}. Beat In = XmIz (Eq. 6.33), and J, (same direction as 1). Kp MX f= XmH xi >| Ky (opposite direction to 1). 2na Solea?) + Ko(2na) = xml = Xm = [0] (as it should be, of course). Problem 6.18 ” By the method of Prob. 6.15: For large 7, we want B(r,) + Bp = Bo2, oH = LB + 1892, and hence W > -2Bor = 2: Bor cos “Potentials” { Wialr.8) = Arr! Pi(cos8), (<2); Woult6) = Bor cosd +3 zB Pi(cos6), (r > R). Boundary Conditions: { (i) Win(R,8) = Wous(R,9), (i) a Ge + HG a = O. (Phe later follows from Eq. 6:36.) (i) > 0 Zersconds Dis nggear(on] + nS tae Pi(cos8) = For 11, (i) => By = R*+4Ay, so (pol! +1) + ulJArR'~! = 0, and hence Ay = 0. 1) ALR = —LBoR+B,/R?, and (il) + Bo-+2aBy/R° + Ay = 0, 80 Ay = ~3Bo/(2p0 +1) 3a Boe 3Bo By Win(r,8) ros 6 = WW. = a on (8) = ase a Bo) = Gin +) ** Banta) — a 3HBo OO Bot i) 1g By the method of Prob. 4.23: Step 1: By magnetizes the sphere: Ma = XmHo = gaciEzcyBo- This magnetization sets up a field within the sphere given by Eq, 6.16: 2 By = 5oMo 2 =p FrBo (where «= 22) 314 Xm Step 2: By magnetizes the sphere an additional amount My = By. This sets up an additional field én the sphere 2 2 26\? Ba = 2yoMy = 2xB, = (25) By, ete. dn dan-(BJm ‘The total field is: Bi B = By +By +Bz +++ = Bo + (2n/3)Bo + (2/3)?Bo +++ = [1 + (2n/3) + (2n/3)* + -+-} Bo = Ty 1 3 349m tym) (ote T= R/S ~ S~PmwMTF Xm)” TFB Bm — BF Hm” P= (TF eel) PO Problem 6.19 . 7 ‘am = ~EEB; M = 4m = - 22,8, where V is the volume per electron. M-= xakt (Bq. 829) = paRcyB (Eo. 6.30). So xm Et yuo. (Note: xm 1, so I won't worry about the (1+ xm) term; for the same reason we need not distinguish B from Baie, as we did in deriving the Clausius-Mossotti equation in Prob. 4.38] Let’s say V = $nr3. Then ym = ~$2 (a) Til use 1 A= 10“ im for r. Then xm = =(10-7 ( ) = be 1077] whieh not bad Table 6.1 says xm = =1 «10%, However, I used only ane electron per atom (copper has 29) and a very erude value for ¢. Since the orbital cadius is smaller for the inner electrons, they count for less (Am ~ r#). T have also neglected competing paramagnetic effects, But never mind ... this isin the right ball park Problem 6.20 ~ Place the object in a region of zero magnetic field, and heat it above the Curie point—or simply drop it on hard surface. ifs delicate (a watch, say), place it between the poles of an electromagnet, and rmagnetize it back and forth many times; each time you reverse the direction, reduce the fed slightly Problem 6.21 ~ (a) Identical to Prob. 4.7, only stesting with Eqs. 6.1 and 6.3 instead of Eqs. 4.4 and 45. (b) Identical to Prob. 4.8, but starting with Eq. 5.87 instead of 3.104 (©) U = - B43 .c08@) cosa — c08(62 — 1)} mimo. Or, using cos(#2 ~ 61) = cos; cos 6, — sin, sin 2, u Stable position occurs at minimum energy: $Y = $f = 2sinby cose, O= 2sinby cose, (e = 4942 (cos, sin B2 + 2sin As cos 2) bos; sin dy; GE = uomT2 (sinG, cosd, + 2cosd sin &) —4.cos fh sin 129 CHAPTER 6. MAGNETOSTATIC FIELDS IN MATTER P+ or ~P— tt o tt ® Which of these is the stable minimum? Certainly not @ or @)—for these zi is not parallel to By, whereas we know my will line up along B,. Jt remains to compare (with 6: = @ = 0) and@ (with & = x/2, 82 = —x/2): Either sin; = sin@, = ‘Thus sin®, cos 8s = sin, cos: = 0.4” or cog9, = costs U, = Sy (—2); Up = #48 (-1), U; is the lower energy, hence the more stable configuration. (Conctusio (a) They'd Tine up the same way: > —> 9 —¥ > —> Problem 6.22 Fas faxp=1(fa) x Bott f axle voype)—1 (fat) tle Vo)Bol (because f dl = 0). Now (d) x Bs), = Dyeaady(Bolay and (Vo) = Dor(Vo)s, 80 am fx (e- Vo)Bo) _ Cts } Tew [fr a} {(Vo)(Bo)ad {tema fe fr a u jet = 1D exjnttzmam(Vo)i(Bo)e. [ten Sr evetsm = didi ~ Sind (proof below! } im 7 = TY Cubem ~ bimbit) dm(Vo)(Bo)a = fas(Wo)lBa)e ~ as(Tole(Boa] = 1{(¥o)ula- Ba) ~as(Vo Bo}] But Vo By = 0 (Eq. 5.48), and on = a (Bq, 8.84), 80 F = Wo(m- Bq) (the subscript just reminds us vo take the derivatives at the point whete m is located). ged Proof of Lemma 1 Bg. 1.108 says f(e-r) dl =a xe = —e xa. The jth component is >, fear dly = p> 5 (ie. 1 for the Ith component, zero for the others). Then $n dls Proof of Lemma 2 cjeetim = 0 unless ijk and Ljm ave both permutations of 123. In particular, # must either be I or m, and k rust be the other, so Em tient Pree m €stmOm ~ Dem Eijmam- ged So eiseetim = Abadi + BEd ij ‘To determine the constant A, pick i=1=1, =m = 3 the only contribution comes from j = 2: Abii53s + Biss = A A=1. ‘Fo determine B, pick i =m = 1, k einen) = 1 = Adiadat + B68 = B > B= —1, Deieeim = badim — Sibir. ged 5 121 Problem 6.23, (a) The electric field inside a uniformly polarized sphere, B= ~ ¢;P (Bq. 4.14) translates to H = ~ 9! (9M) = =1M. But B = jio(H+M). So the magnetic field inside a uniformly magnetized sphere is B= juo(—1M-+M) = FroM| (same as Ba, 6.16) (b) The electric fed inside a sphere of linear dilectric in an otherwise uniform electric field is = yq27yEo (Bq. 449). Now xe translates to xm, for then Eq. 4.30 (P = eoxeE) goes to uM = HoXmH, or M = XmH (Bq. 6.29). So Bq. 449 > He = Lata. But B= yo(t + Xm)H, and By = HoH (Kqs. 6.31 and 6.32), 0 the magnetic field inside a sphere of linear magnetic material in an otherwise uniform magnetic field is — B= 1 Be ip = (A*%) | (as in Prob, 6.18) Bata) ~ TF xa ns? |B (TSG) Bo] sn Ped 8) (0) The average electric field over a sphere, due to charges within, is Byye = — zfs. Let's pretend the charges are all due to the frozen-in polarization of some medium (whatever p might be, we can solve V-P = ~p to find the appropriate P). In this case there are no free.charges, and p= [P dr, 80 Eave = ~ aia; ay PAP, which translates to But B = po(H + M), 50 Bure = —£298 + HoMaye, and Mave = thr, 50 | Bave in agreement with Eq. 5.89. (We must sssume for this argument that all the currents are bound, bat again it doesn’t really matter, since we can model any current configuration by an appropriate frozen-in magnetization, See G. H. Goedecke, Am. J. Phys. 66, 1010 (1998),) Problem 6.24 Fq.215: BE = p{zh; Adv} (for uniform charge density); Badg: v= P-{z fy dar} (for uniform polarization); Bq. 61: A = pocoM x {<2 fy Sede} (for uniform magnetization) Ein size) (Prob. 2.12), For a uniformly charged sphere (radius f) Ne ) : : ot = p(s) (Ex. 22). Va = gi (P-r) An = @(Mxr), (confirming the results of Ex. 4.2 and of Exs. 6.1 and 5.11). Problem 6.25 @) By = 8298 (Eq 586, with @=0). Som,-B, = ~H2. F = V(m-B) (¥a.63)4 F=f [Be ‘Yams. This is the magnetic force upward (on the upper magnet); it balances the gravitational force downward mage): 122 CHAPTER 6. MAGNETOSTATIC FIELDS IN MATTER (0) The middle magnet is repelled upward by lower magnet and downward by upper magnet: ‘The top magnet is repelled upuerd by middle magnet, and attracted downward by lower magnet: Som? Spot ny? Ine ty) mag =0. cart amar 350115... Subtracting: 2492 [Je ~ fe ~ ge + petige] -mag-tmag = 0, oF Ze— fet gape = 0580: 2 Let a= 2/y; then 2 = 2r+ gtr. Mathematica gives the numerical solution a =[2/y = Problem 6.26 = At the interface, the perpendicular component of B is continuous (Eq. 6.26), and the parallel component of His continuous (Eq. 6.25 with Ky = 0). So B} = B}, H! = Hi, But B = uH (Eq. 6.31), so 2B] = 2 Bl Now tan, = B!/B}, and tand, = B!/B}, so tan, _ Bi Bl tant BE Bf Bl (the same form, though for different reasons, as Eq. 4.68) Problem 6.27 Tn view of Eq, 6.33, there is a bound dipole at the center: my = xm. So the net dipole moment at the center is meenter = M+ my = (1+ Ym)m = Am. This produces a field given by Eq, 5.87 Beige = 24 dipole dr? (Sm =n. ‘This accounts for the first term in the field. The remainder must be due to the bound surface current (Ks) at 1 = R (since there can be no volume bound current, according to Eq. 6.33). Let us make an educated guess ‘oased either on the answer provided or on the analogous electrical Prob. 4.34) that the field due to the surface bound current is fr interior points) ofthe form Beyrace = Am (Le. a constant, proportional tom). In that ‘case the magnetization will be: Xmp — Xm 1 ee M = Xnb (mn #)# — mn} + XA, (3(m.#} a + ri This will produce bound currents Jy = VXM = 0, as it should, for 0 R) is rat (asmea trae B= (gta) Jot #)e — m) Broviem 638 ‘The problent is that the feld inside a covity is not the same as the field in the material itself. (a) Ampére type. The field deep inside the magnet is that ofa long solenoid, Bp ~ 4oM. From Prob. 6.13: fo { Sphere: B= Bo ~ }4oM = 540M; Needle: B= Bo ~ jioM Wafer: B= 10M. (b) Gilbert type. This is analogous to the electrie case. The field at the center is approximately that midway between two distant point charges, Bo 0. From Prob. 4.16 (with E> B, 1/¢o > Ho, P > M): { Sphere: B= Bo + 4M = SuoM; Needle: B= Bo = 0; Wafer: B= Bo + 0M = 10M. In the cavities, then, the fields are the same for the two models, and this will be no test at all. [Yes.] Fund it with $1 M from the Office of Alternative Medicine, Chapter 7 Electrodynamics Problem 7.1 (2) Let @ be the charge on the inner shell. Then E = ;3~ $7 in the space between them, and (Vs ~ Va) = = BBete= ake ff har = 85 (2D oe fedex o8 = 2 ntsl¥e- Ve) We [temo [etenod = 5 it - [ae tae 11). () R= (2 iy] T=[ae (a7) | (©) For lange b (8 > a), the second term is negligible, and R = 1/4rca, Baccnally all of the eistance isin the region right around the inner sphere. Successive shells, as you go out, contribute less and less, vecause the cross-sectional area (4xr?) gets larger and larger. For the two submerged spheres, R= z2- = sil; (one Ras the current leaves the fist, one Ras it converges on the second). Therefore = V/R=[2 Problem 7.2 ~ ~ (@) V = Q/C = IR, Because postive I means the charge on the capacitor is decreasing, a 1 ‘ine 7p, 0 [Q{H) = Ove" | cl FEA © Ul) = QoeH!F°, But Qo = Q(0) = C¥o, 90 [Q(t) = C¥oe™*/** | Hence I(t) = -4 [% (0) = [ei] he enya oe storie [Pe Vg (_RC ra YE (_RC_-njac\|” _ Loya ( ~ )f 3eve. v a [Crna= 8 [rma 1 cw poedeee) REO Boy = at = In(Q— CV) = — Fat + constant + Qlt) = CV + hew"/ME. But Q(0) = 0 > & = ~C¥o, s0 126 CHAPTER T. ELECTRODYNAMICS ngs tom batons: [rat = ME [ eecar — (-rcerv0e) | (a) Energy from battery: [vor at = [™ e-wean = 48 (-rcerv ee) Since [(t) is the same as in (a), the energy delivered to the resistor is again “xo =[ova, CV§.| The final energy in thie capacitor is also [TCG] so [half] the energy from the battery goes to the capacitor, and the other half tose eestor. Probiem 7.3 (a) 1 = [J-da, where the integral is caken over a surface enclosing the positively charged conductor. But J = oF, and Gauss’s law says JE-da = 2Q, 90 1 =o fE-da = £Q. But Q = CV, and V = IR, s0 = £01R,o[n= 2] aed (b) Q@ = CV =CIR = 4B = -1 =~ feQ | At) = Que“ /"° |, or, since V = Q/C, V{t) = Voe“#/FO. The time constant sr = RC =[eole] Problem 7a T= J(e)2eab = J(s) = I/2x0b. Bs JJo = [)2xeab = 1/2nkl. ye- [rs a Ve eee a aED Froblem 75 é a ER dP veri PPR Grape aR =). So Serta Rs Gee Problem 7.6 : Edi = [z270] for all electrostatic fields. It looks as though € = $ E- dl = (o/eo)h, as would indeed bie the case if the field were really just 9/9 inside and zero outside, But in fact there is always a "fringing ficld” at the edges (Fig. 4.31), and this is evidently just right to kill off the contribution from the left end of the loop. The current is [72F0.] Problem 77 (@) € =~ = -pif Biv; € = IR (Never mind the minus sign—it just tells you the direction of flow: (v x B) is upward, in the bar, so downward through the resistor.) (o) P= 18 As (0) Fem |v = wer Rt (@) The energy goes into heat: in the resistor. The power delivered to resistor is PR, so a, BPE gate, wrereas 2, H— mage-ot aa BE gem, whmeas AE, Me amage The total energy delivered tothe resistoris W = amug fe ?tdt = amug 17 (7 Problem 7.8 (a) The field of long wire is B= MO! 6, 50 & [ea a= tat [too [0 EE). Ins or Js A (S ao ale dg (2 _ a st ds gy mola (_1_ds_1ds\_{ pole wee P= S ge (42) am Gone 98 ag Ga) [mera ‘The field points out of the page, 30 the force on a charge in the nearby side of the square is f0 thé right, In the far side it's also to the right, but here the field is weaker, so the current flows | counterclockwise. (6) This time the lux is constant, so [E Problem 7.9 Since V-B boundary tine, |, Theorem 2(c) (Sect. 1.6.2) guarantees that fB-da is the same for all surfaces with a given Probiem 7-10 © =B-a=Bo?cosd — Here @ = wt, so B ame i (view from above) Bo?(~ sinwt) Problem 7.11 €=Bly downward: 2». This opposes the gravitational force y= upward magnetic force = IIB dy Be Gaon aw, where a=. g—au=0> 4 At 90% of terminal veloc t ~at; In 10 = at; 1n10, of | tang, = “E10 10, wom =F Now the numbers; m= 4qAl, where 1 is the mass density of aluminum, A is the cross-sectional area, and Lis the length of a side. = 4I/-Ac, where o is the conductivity of aluminum. So p= 28x 10-° 2m AnAlgAt “= Aoeer T toon = LEW in(.0) = [28 ms. fF the loop were cut, it would fall freely, with acceleration 9. $0 vy = OMe sYe.tatoy2.8-10-% 128 cc Problem 7.12 o=n(%)'B 2 TE conus) €= HF = ™ Byusin(et). 110) = CHAPTER 7. ELECTRODYNAMICS db _ ra! rate =| Tp Bo sin(ut). palo Problem 7.13, a= [reas [ee [parm thee. €= Problem 7.14 pipe—-| falling __| magnet Suppose the current (1) in the magnet flows counterclockwise (viewed from above), as shown, so its field, near the ends, points upward. A ring of pipe Below the magnet experiences an increasing upward flux, as the magnet, approaches, and hence (by Lena's law) a current (Jiqq) will be induced in it such as to produce a dounward flux. Thus lina must flow clockwise, which is opposite to the current in the magnet. Since opposite currents repel, the force on the magnet is upward. Meanwhile, a ring above the magnet experiences 1a decreasing (upward) flux, so its induced current is parallel to Z, and it attracts the magnet upward. And the flux through tings nezt to the magnet is constant, so ne current is induced in them. Conclusion: the delay is due to forces exerted on the magnet by induced eddy currents in the pipe. Problem 7.15 In the quasistatic approximation, B = { r a wont, {8 a). Inside: for an “amperian loop” of radius s .a: & = Bra? yon? dl a 2s dt pontna®; Ens = —yonna® Problem 7.16 {a) The magnetic field (in the quasistatic approximation) is “circumferential”. This is analogous to the current in a solenoid, and hence the field is [longitudinal (b) Use the “amperian loop” shown. 0, so here B= 0 (like B outside a solenoid. = Beda el, Outside, So fEvdl= Bl = #240 In (3) sof = #2! sot) n (2) But = = —Iowsinwt, co ie oo 129 Problem 7.17 (a) The field inside the solenoid is B = onl. So @ = na®yonl = € = —x0 pon(al/dt) In magnitude, then, € = xa2jonk, Now £ = I-R, 0| Iresistor = 7 pont B is to the right and increasing, 90 the field of the loop 1 to the lef 0 the current is counterclockwise, or to the right, through the resistor. oi oo Dra pont AQ= FAG, in magnitude. So [ag = “A tent, () Ad = 27a ponli woaln2 dt “a Oe de” Problem 7.18 a= [0 The field of the wire, at the square loop, is out of the page, and decreasing, so the field of the is current must point out of page, within the loop, and hence the induced current flows Problem 7.19 ant} In the quasistatic approximation, B = {e oe (Bq, 5.58). The flux around the toroid is therefore juced (inside toroid); (outside toroid) Qa dt ta di Dna . BN Vay = HONE n(1¥2) woNhw, d® _ poNhw al _ toNhwk ‘The electric field is the same 5 the magnetic field of a circular current (Eq. 5.38): with (Bq. 7.18) py _1dd __Nhwk -2( Nhwk ; faa tre” SE Fara) eran? i _Nhwke_, + PP Problem 7-20 (a) From Bq. 5.38, the field (on the axis) is B = 44 Ge" rs7ed, so the flux through the little loop (area wa*) a worl’? (i + (b) The field (Eq. 5.86) is B = 42 9 (2cos6# +sin6), where m = Ina?. Integrating over the spherical “cap” (bounded by the big loop and ceniered at the litle loop): a te, f° cos sind b= [peda= BEE [ercosaye? sin@ d0.d6) 130 CHAPTER 7, ELECTRODYNAMICS 2 intel? where r= VFF2 and sind = b/r. Evidently @ = Halts! sista . = 7 M2 none (0) Dividing off 1 y= Miala, Bz = Mer): [ha = Ma = sop ae Probie TH a a MG Me le It’s hard to calculate M using a current in the little loop, 59, exploiting the equality of the mutual inductances, T'll find the flux through the little Joop when a current J flows in the big loop: = MJ. The field of one long wire is B= $2! = 8, = #2! J" Lads = Hl? ina, so the total fux s pafain _ garb, = Halen? , yy monn? in magnitude Direction: The net Hux (Uhrough the big loop), due to Fin the litle loop, is into the page (Why? Field lines point in, for the inside ofthe little loop, and out everywhere ouside the litte Toop. The big loop encloses all of the former, and only part of the latter, so net Aux is snuard) "This fux is increasing, go the induced current in the big loop is such that is eld points out of the page: it Rows [counterclockwise Problem 7.22 B = pon! = 4, = ponte? (fox through a single turn). In a length 2 there are nl such turns, s0 the total ux 6 = won eR'ML. The selfinduetance is given by @ = LI, 30 the sel-inductance pet unit length is = porta Problem 7.23 ; The field of one wire is By = $24, so = 2. mh -1'f = lin (454). ‘The € in the numerator is Jigible (compared to d), but in the denominator we cannot let ¢—+ 0, else the flux is infinite ee tol In(d/e)|- Evidently the size of the wire itself is critical in determining Z. Problem 7.28 a (o) Inthe quasistatie approximation B= £23, So, = 4 f° Ende = Hal inc) Ni ‘This is the flux through one turn; the total fux is N times &: = MN in(a/a)ia cos(ut). So 2 4 _ oN (4 « 10-7)(10°)(0~ 6 =F = ONE neyfoyfavsin(ot) ae 1n(2)(0.5)(22 60) sin(ut) aaa _ _ _& _ 261 x10-¢ = [2.61 x 1074 sin(ut)] (in volts), where w = 29 60 = 377/s. Ir = Fy = oy — sinlwt) = [5.22 x 10-7 sin (wt) | (amperes). (b) & =~ 5 where (Eq. 7.27) L= e9Q228 jn(bfa) = 42*10-7)120")00"*) jn (9) = 1,39 x 1078 (hensies). ‘Therefore € = —(1.39 x 107*)(5.22 x 1077 w) cos(wt) = [=2.74 x 10-7 cos(ae) | (volts). 131 2.74 x 10-7 Ratio of amplitudes: F220 = [1.05 x 1 Problem 7.25 ‘With I positive clockwise, £ = -L4! = Q/C, where Q is the charge on the capacitor; I = 42, so £2 = —1hQ = -w°Q, where w = yhg. The general solution is Q(t) = Acoswt + Bsinut. At t = 0, Q=CV, s0 A= CV; I(t) = 9 = —Ausinut + Businut. At t=0, 1 =0, so B=0, and creas Em (Gi) If you putin a resistor, che osilation is “damped”. This time -L4f = 9 +1R, so L494 R42 +3Q For an analysis of this case, see Purcells Blectrcity and Magnetism (Ch. §) of any book on ostillations and waves Problem 726 (@) W = BLP. L= pon?a RPL (Prob. 7.22) |W = yon PUP (b) W =} §(A- Dal. A = (wont/2)R4, at the surface (Bq, 5.70 oF 5.71). So Ws = 54 RI-2eR, for one turn, There are nl such turns in length I, 30 W = hyon®x REL, ¥ (©) W = pz fB%dr. B = ponl, inside, and zero outside; far = xR, so W = ouin?l?eR't = Suon®a RPT. v (a) W = ph; [[ BP dr — §(A x B) da). This time [B? dr AXB =O outside (at 6 = 5), Inside, A = Mad (at AX B= Lidn® Tag x ) a(t) 13n?P?n(R2 — 02). Meanwhile, @), while B= gn! 2 points inward (*out” of the volume) : . £ §(A x B) -da = Uhugn* a8) -jadp de(—a)] = “hug? PaP2e, — —— ip W = ahs [pbn2 a(R a2) + un? ?xa*l] We Problem 727 = - = = pont, -al Pdr = 1 pemil? B= Fe Y= Ig | P= ig ae? z= Hatain(e/a)| (came as Ba. 727) Problem 7.28 = — Bs h= D(2rs) = polne = pal(o7/R*) > B = HFS, 1 BP fF a wll 548 pol 2 _ 1, paper Peanytds = TaD = get = ee Ho/8r,] independent of R! Ht Problem 7.29 dt at (a) Initial current: fy = &/R. So -L5, = IR3 F ee [oma % ( 132 CHAPTER 7. ELECTRODYNAMICS (6) Wo = $L13 = § (fo/R)”. Problem 7.30 (a) By = 2 H(A) fay. ‘The flux through loop 2 is then ~ ay}, since m: $= By a= BE nila Yan) ayaa] = Mh. [M = 28-190 8)(aa +4) — 1 - a (0) & =—83, ME), = Eh = Mh 9R. (This is the work done per unit time against the mutual emf in loop 1—hence'the minus sign.) So (since It is constant) Wi = MIif2, where Iz is the final current in loop 2: Ww = ASfatm: «(m2 -4) ~ my «me ‘Notice that this is opposite in sign to Eq. 6.35. In Prob. 6.21 we assumed that the magnitudes of the dipole moments were fied, and we did not worry about the energy necessary to sustain the currents thersselves—only the energy tequired. to move them into position and rotate them into their final orientations. But in this problem we are including it all and it isa curious fact that this merely changes the sign of the answer. For ‘commentary on this subtle issue sce R. H. Young, Am. J. Phys. 66, 1043 (1998), and the references cited there. Problem 7.31 The displacement current density (Sect. 7.3.2) is Je radius s, = shy #. Drawing an “amperian loop” at = Hols? pols B= Freat) [B= ano Problem 7.32 wes Os, 2 2 7 0) Hane = Jan? % fe = Holige > B2R8 = pol > B =| Hag, () A surface current flows radially outward over the left plate; let 1(s) be the total current crassing a circle of radius s. The charge density (at time ¢) is _ =H = Ho a(t) Since we are told this is independent of s, it must be that J ~ [(s) = 8s, for some constant 9. But I(a) = 0, 50 fia? = I, or f= I/a?, Therefore I(a) = 1(1 ~ s*/a"). 2 es Bins = Hole = poll 110) = woop 9 [B= Es4.] v Problem 7.33 ae Holo (a) J. ae cos(ut) In (a/s) 2. But Zos0s(wt) = I. So B90 HEAT In(a/s)2 ()le= [ae-aa= Meet ff viotoyansds) = neo [(stoa~sinsyés Sino + 2] = wacow*s [Gta gia +] =| Howat = poeou"t [(Ina) $ — Since poss = fe, La/I = (va/2c)*. 1a = 10% m, and # = abs, so that $2 3G = BARE, or w = 0.6 x 10!/s =[6 x 10°? /si] v microwave region, way ehove radio frequencies) Problem 7.34 Physically, this is the field of a point charge ~@ at the origin, out to an expanding spherical shell of radius vt; outside this shell the field is 2ero. Evidently the shell carzies the opposite charge, +q. Mathematically, using product rule #5 and Eq. 1.99: © 10!°Hz, os 104 megahertz. (This is the 8) Sou v Beaut-nv:(-2 4 ‘)- Se Vie(ot—r) as LHe) “et But Hr}alut 2) = 6(e)0(e), and Zo(ut ~) = ~6(et — +) (Prob. 1.48), 50 p= aV B= |—a6%(r)0@) + ») (For t <0 the field and the charge density are zero everywhere.) Clearly V -B = 0, and V x B = 0 (since E has only an r component, and it is independent of @ and ¢). ‘There remains only the Ampére/Maxwell law, V x B= 0= ol + s¢o9E/0t. Evidently wots een} «= [Fes] (The stationary charge at the origin does not contribute to J, of course; for the expanding shell we have J = pv, as expected—Eq. 5.26.) _ Problem 7.35 From V-B = jopm it follows thatthe field of a point monopole is B = $434. ‘The force law has the form F 6 dm (B- ev xB) (see Prob. 5.21—thw cis needed on dimensional grounds). ‘The proportionality (2 over the surface of the loop: a 7 a “mofo 4 fav b= —pohnge constant must be 1 to reproduce “Coulomb's law” for point charges at rest. So Problem 7.36 Integrate the "generalized Faraday law” (Bq. 7.43iii), V x E = —HoTm Jivxv-e- fra But € =~. a 50 g in + 48, or = BAQn + ZA#, where AQm is the total magnetic charge passing through the Surface, and Ad fe the change i flux thigh the surface, I we use the fla uit then AQm = Gm and A® = 0 (when the monopole is far away, & = 0; the flux builds up to pogm/2 just before passes through the loop; then it abruptly drops to ~o%m/2, and rises back up to zero as the monopole disappears into the distance). If we use a huge balloon-shaped surface, so that gm, remains inside it on the far side, then AQz, =0, but & rises monotonically from 0 to jdm. In either case, e Hates E L 134 CHAPTER 7, ELECTRODYNAMICS Problem 7.37 pipe YOR 8 [Vocos(2nvt)] _ Vo, ony, Bata hncb= teat yn x | oavsin Ore}, ‘The ratio ofthe amplivudes is therefore Je _ Vad 1 [2n(4 x 108)(81)(8.85 x 107 )(0.23))* Ta pd teves ~ Tavs Problem 7.38 ‘The potential and field in this configuration are identical to those in the upper half of Ex. 3.8, Therefore: 1 / Jeda=o f Eda where the integral is over the hemispherical surface just outside the sphere ——w _~ But I can with impunity clase this surface: (because # = 0 down there anyway—inside a conductor), Sof =o [E-do = £Qene = £ Je da, where or is the electric charge density on the surface of the hemisphere— to wit (Eq, 3.77) a2 = SeoBy ccs. 5 zi 1 Eeoby fcosta?sin 848 dp = So Bya?2x "sin cost dd = aeons? But inthis case By'= Vo/d,so| f= S2*¥0 Problem 7.39 Begin with a different problem: ¢wo parallel wires carrying charges +A and —) as shown. Field of one wire: B= -48; potential: V Bar ‘rae In(s/a} Potential of combination: V = zA-In(s_/sz), cain { nes} Find the locus of points of fixed V (ie. equipotential surfaces): were? Quo or Viys2) = efro¥)d = => ply? — 2yb +0 + 22) = y? + Byd 4 42s PHD) HUY =0 e+e DH =0 (9= (6? - 1). (y— bB) + 2? +8? — 886? <0 => (y 00) +2 7 This is a circle, with center at yp = 69 = b(t) and radius = bY =I = by HERE ae ‘This suggests an image solution tothe problem at hand, We want gp = radius =a, and V = Vo. ‘These determine the parameters 2p, and A of the image solution: stall ae a cats eee tae + (2-40°)u sl = 2a? 1 V1 dat + dad 1 = 20? - 1+ 20a 1; snes i ‘That's the line charge in the image problem. in Qa? 14 2aVoP—1)" loco 2 [ieteme [dan oi Qene= 2 eee T>@ * naar the cylinders are far apart, d % a, so that a > 2. (= 208-1202 /i=Tfe? = 20? 120% [1 Which sign do we want? Suppose ] nda? (+ sign), (= sign), ‘The current per unit length is { = 2o%(1 y-wane graf 7 The current must surely decrease with increasing a, so evidently the + sign is correct Problem 7.40, (a) The resistance of one disk (Bx. 7.1) is dR. disk, ‘The total resistance is +h aie, sixdz, where r = (154) 2 +a is the radius of the [ wea aera tal (b) In Ex. 7.1 the current was parallel to the axis; here it certainly is not. (Nor is it radial with respect to the apex of the cone, since the ends are flat. This is not an easy configuration to solve exactly.) (©) This time the flow is radial, and we can add the resistances of nested spherical shells: dR, A -f[ 7? sin Bd dd = Inr(— cos8)|é = 2n13(I ~ cos). EE 5 o qa where A : 136 CHAPTER 7. ELECTRODYNAMICS _ e__ =P (B=) Now 2 eb sin R= Se ead) £ I" = Fr —cos8) ( rare es re in? = Mena) Sine ut sin@= PA and cond = = Bat sind = eters and cos = PES Pee tt ee SORE fi SOE 2, asin (a) Problem 7.41 Vinod) = So stbysin(kg), (8 a). (We don’t need the cosine terms, because V is clearly an odd function of ¢.) At ¢ =a, Via = Vout = Vob/2m- ae ’s start with Vig, and use pe 's trick to determine bk: EE Setasinte = =e yen ie sin( kg) sing) ap = 32 ef dsin(&’0) dé. But r sin(kd) sin(R'@) dé = m5, and [ dsin(k'§) dp = lernwo - Sootes = ~Reos(h') = -FE(-1)". $0 ate aC) . and hence Vin(s, é) on ne eoen 2 = sfaforr VT (gamey') (et scod = |(aF v0089)? + (esind) _ Yef__asing =} (aniteere)- ave sing (eka) mo = Bae Problem 7.42 (a) Faraday’s law says Vx = ~ $2, s0 E = 0=> 92 () Faraday's law in integral form (Eq, 7.18) says f E-dl the loop is constant. (0) Ampére-Maxwell-> VB = pod + noto9E, so E = 0, B =0= J =0, and hence any current must be at the surface. (@) From Eq. 5.68, a rotating shell produces a uniform magnetic field (inside): B = }uowat. So to cancel 3 Bo : 3B nog ee 3h 08,50 = 2 sina = B(r) is independent of t —d@/dt. In the wire itself E = 0, so ® through such a field, we need owa = Problem 7.43 (a) To make the field parallel to the plane, we need image monopoles of the sume sign (compare Figs. 2.13, and 2.14), $0 the image dipole points [down (2).. (b) From Prob. 6.3 (with r -> 22): Sag rat Po oe ae 138 CHAPTER 7. ELECTRODYNAMICS (©) Using Bq. 5.87, and referring to the figure: m la p=% [S(m & - #1) #1 ~ me] + [3(-ma- fa) fe + mg] eles 2 apt 1) By = mi 4-2) a + ma} = ie 7 a iM = = eRe IADR ~ (ots) fa]. But a. = a -fy = ost a HOM (F, + F; jut # +F = 2sinee. an = Sin een But f +i =2sinos. a7 = 3H! co uA - Se lo ~— BIST sindcosde. Dut snd= 2, cost =, andi = VFR . Siomh__r or aye Now B= p(k x8) 9 8D = job x (K <2) = IK ~2-2) = oR. (Posed he BAGOAD al, and noted that K-# = 0, because the surface current isin the 2y plane.) 1 3mh _3mh__ot Jew) = -34 tex ae Ka Ox 8) = SE pp = aaah aed Problem 7.44 ~ ~ ~ 7 a Say the angle between the dipole (m;) and the 2 axis is @ (see diagram). Lt ‘The field of the image dipole (mz) is “™ 1 oa 2) = Gt sp Blom H)A— sh - . la for points on the = axis (Bq. 5.87). The torque on mi is (Ea. 6.1) Nam xB= tary em a)(m, x 2) — (my x ma)}. anh But m, = m(sin 8% + cos02), mz = m(sin@& ~ cosa), s0 mz = ~mos8, mx 2= my x my = 2m? sin cos 69. bom? oon y= a 4x (2h)> Evidently the torque is zero for @ = 0, 7/2, or m. But 0 and x ase clearly unstable, since the nearby fends of the dipoles (minus, in the figure) dominate, and they repel. The stable configuration is @ = "/2: parallel to the surface] (contrast Prob. 4.6). Tn this orientation, B(=) = ~ five and the force on my is (Eq. 6.3): rev attra cae [am sin @cos@9 ~ 2m sin 6 cos89)) sin 0.080. _ om? ~ arpa At equilibrium this force upward balances the weight Mg: ae Sugm? =| (Suom’ anny M9 "= 15 aentg) 139 Incidentally, this is (1/2)"* = 0.84 times the height it would adopt in the orientation perpendicular to the plane (Prob. 7.430). Problem 7.45 7 vx By =wasindd; f so? By J sin (6 x @ “A Ex wea [ sin 9 c0s0.d0 = wa? Bo and dl = ad96 (xd) (same as the rotating disk in Ex. 7.4) waBysin(6 x#). a0. But 8-6 x a sin! Oy y9/2 z Wo =| +B = cos8, oa? B, Problem 7.46 (a) In the “square” orientation (Q), it falls at terminal velocity | [Pea (Prob. 7.11). In the “diamond” orientation (©), the magnetic force upward is F = [Bd (Prob. 6 0 90 = Bl? — (I/V2 - y)?}. ~2B (l/v2—y) $f. But $f = -v. 2Bu (Ui V2 ~ y) = IR => I = 2B" (ly V2 ~y); F = 2-28 (I/VO-y) (This works for negative y as well as positive, if you replace y by [yl.) mg oe terminal velocity). Haas. (meh) 4 v2-v) a me = 2y/i) «| Avsist (y ~ Soe THs mond (BE mgt (v2—2ust) -| At first (y ~ 1/2) the “aiamond” falls faster; toward the halfway mark (y ~ 0), the “square” falls twice as fast; then the diamond again takes over. The total time it takes for the square to fall is: Be Pquare | mOR fequace (assuming it always goes at the terminal velocity, which—as we found in Prob. 7.11—is close to the truth, if the field is strong). For the diamond, t is : a dy 887 2 8B ie BBP 22 BS - fH 8 fy) q — see ya Be Soancos = mak (uaa) r= an [5 UY Vee mgR3293 | 3 mah ie $0 tigare/taiamond = 3/2V2 = 1.06. The “square” falls faster, overall If free to rotate, it would start out in the “diamond” orientation, switch to “square” for the middle portion, and then switch back to diamond, always trying to present the minimum chord at the field’s edge. * (b) F = 1B, & =2Bf, Jaa dz (a = radius of circle) = 48 = -ap Ye? = 2Bvfa =H = IR KA BE Vie) = aS =F 80 F = Ce =) = mg nT ght sa = Ga = 4B ast ; LULL 40 CHAPTER 7. ELECTRODYNAMICS Problem 7.47 (a) In magnetostaties V-B=0, VxBayos = Bey = M2 | 4 ae For Faraday electri feds (with p = 0), therefore, 3 ow 1d (BED xs, weban vxpe 2 ow Bese 22 [Beni (with the substitution J > —3-9.) (b) From Prob. 5 50a, (Check: Vx E {) The Coulomb feld i zero inside and 72-9 # £8 f outside. The Faraday field is 94, where A is given (in the quasistatic approximation) by Hq: 5.67, with a function of time. Letting & = da/dt. phir nad (re), 5.040 oF, wie snd o>n| Problem 7.48 “nw = = mv is to stay fixed, then gR2E = m%& 2 ma = F = qB, or B= RO. Bu aBR = mu (Eq, 5.3). If Ris to stay fixed, then gh =m F = qb or B= RO. But a a 1 db dB 1/4 d= 8, Bah = -B, oo ~ 1M = eB, or B= -} (a) + constant. an vine t= fed=—-&, opine =, so ~ nG, ora =-4 (Foe) + constant. artnet =0 th fa of hn ee conta so and 8) = 2 (Le) Gm mag), Edy he eta must be half the average field over the cross-section of the orbit. “ged, Problem 7.49. Tnitially, ®% = ch-19 > 7 = Lym? new ofbit, of radius r1 aid velocity 0, aiigi@. After the magnetic field is on, the electron cixcles in a mj 1 = $42, or mdu = £4B. ‘The increase in the two expressions conclude that Mi Problem 7.50 €=—%% = ~a, So the current in Ri and Rais [= G by Lena's law, it flows counterclockwise. Now Rtk the voltage across Ry (which voltmeter #1 measures) is Vi = Ij (Vs is the higher potential), Take and Vp= 12 =] FS) (Wis lower). Problem 7.51 ao, p= fy _ nat wi Es uBh=-LGh P= InB = mG, = SS | Problem 7.52 A point on the upper loop: r2 = (a¢0s a, asin da,2); @ point on the lower loop: r1 = (bcosgi,bsin 0). a? = (rp — 11)? = (@c05 2 ~ beosgy)? + (asin gy — bsin gy)? + 2? = 2? cos? $x ~ Daboos a cos fy + 8 cos? fy + 0° sin® go — 2absingy sina + 0° sin? + 3? =a? +67 + 2? — 2ablcos dy cos dy oe =@ + +2? — Qabcos(Gr — 1) = (a? +8 +2)[1 ~ 26 cos(@ ~ 1)] = Bla ~ 20 cones - o)]) dda by = adgo[— sin br X + c08 $2 ¥], 80 abcos( ba ~ #1) dir den. = - Ban | pes ~ OL bdgy[—sin $y & + 0056, J]; dle ‘abidpy dea[sin dy sin bn + £05 6, c05 6 dy do Both integrals run from 0 to 2r. Do the q integral first, letting u = tem f wiim-[ me (since the integral runs over a complete cycle of cos u, we may as well change the 41 integral is just 2x, and a8 [ to 0 2z). ‘Then the M = 2 yabpon f 7a (2) If ais small, then «1, 0 (using the binomial theoret) 1 an a 2 ers + feos, and — az [ cose +8 [ cos? udu = 0+ Br, moa?d® and hence M = (uor/2}/0B. Moreover, 0 % bf (+2), so M | pPTT | (ame asin Prob 7.20). 2 CHAPTER 7, ELECTRODYNAMICS (b) More generally, =D oy as geass Spt eost + Sp cast (+e > FES TTSRE Tt Boost SP cos? ut FOP cost ut, 0 ae Bf coset cos?uan + 56° cot utes 56? [ cost udu + } 2 I, 2 2 j, = 2 abi o + ae) +$ S80) + 308m + | 98 Vans (1+ Bor + (ya +--) aed Probiom 7.53 Tet # be the fu of B through a single Top of either col, so that = Ny and & = Na®. Then & No Rom Problem 7.54 {a) Suppose custent y flows in coil 1, and J in coil 2. Then (if @ is the flux through one vura): M_ pls Mh t+Mh=M% O=hlt+Mh=M%, we=h ment Tn case a =0, we have # = fy if =O, we have 4 = gh. Dividing: #4 = 44, or Lila = (b) Ey = 9p = Lith 4 Map = Vj coslut); -&p = a = Eg" + MEL = BR. god oO ei the first equation by Lj: LyLa i sty = EnV, coswt, Plug in an -hR- Mt 1M nade = taonat |) = oma p+ (bsnl) = Homa SEH (coat Bosinat) [00 = 7 Tamas Bcosut). ce) Ma = BR = Eco Le 8 ane aio f the amptindesis 2. ged () Pe = Vash = (Vi cost) (#4) ( sinut + 2 coset) S a (Esmutcosut + Beasut). Poe = Vowel = ()°R = 80 (P, cos? ut.| Average of coe! wt is 1/2; average of sinwt coswt is zero. MR = 30069 (FR) 5 Pod = Bova? [SSE] |r = (Poo Problem 7.55 (a) The continity equation says $f ot) Putting in the r dependence explicitly, and noting that V--J. Here the right side is independent of t, so we can integrate: (— V-J}t+ constant. The “constant” may be a function of it’s only constant with respect to t. So, ~A(r,0), ale,t) = Ale. O)t-+ ales). ged rc 143 (b) Suppose E = z2-- sea dr and B = 42 [234 dr. We want to show that V-B = 0, VXB = Hod + Hot 9; V-E= Xp, and VxE = -2, vided that Tis independent oft. ‘We know from Ch. 2 that Coulomb's law (B= zh; {$#adr) satisfies V-E = 2p and VxE = 0. Since Bis in time), the V-B and V x E equations are satisied. From Chapter 5 (specifically, Bas. 5.45-5.48) we know that the Biot-Savart law satisfies V-B = 0. Tt remains only to check VB. The argument in Sect, 5.3.2 catres through until the equation folowing Eq, 5.52, where I invoked V'+J = 0, In its place we now put Viele dr (qs. 5.49-5.51) (Capvy (4.5.52) Integration by parts yields two terms, one of which becomes a surface integral, and goes to zero. The other is BV t= 302). So a 1 pa OE YXB = jo3 ~ [3 Pdr = pod +1005 (ae Gar} bod + 10605. ged Problem 7.56 1 (-A)de @) ae = jas VEER ede = es reo sind 1 A —— Bol (gece gasses: k VORoF- vor s [Ver Fa — VEE S - (cut) + (vt)] © neat o[A{_we ato, a Ode Ja~ira Jere As €-+0, vt < € also 40, 90 [a 3(2u) = Av = I. With an infinitesimal gap we attribute the magnetic field to displacement current, instead of rel curren, but we get the same answer. qed Problem 7.57 cv = 22 (Mel) «BD 2 (EL) 09 £ (of) <0. of Ato romtany = df J = Aln(s/s9) (so nother constant). But {i) + {(2) = 0, $0 In(®/s) = 0, 2029 = by and 144 CHAPTER 7. ELECTRODYNAMICS be ab mao | Tp2 talo/0) Ioz_1__, Io n(s/6) ra? sin(ajb) ” * za? In(a/b) (0) o(2) = 60 [B.(a*) - Ba) = eae 2) -4 Sey Problem 758 i saint G [fo fh 4 h 1 le (a) Pasalloplate capacitor; B= Zo; V = ara 2Sns0=8 e= 8 (0) B= wok = nok; 6 = Bri = Mh = LL = Hh (0) [P= toe] = (4% 10°7)(685 x 10-9) = [712 x10" al (Propagation speed 1/ VEC = 1/ /floe = 2.999 x 10" m/s = ¢.) @ D=08 fo s0}us replaces by YES H=K, LK, so just replace jug by ¥. Brobiem 750 (a) J = 0(E+v x B); J finite, o = 00 E+(v x B) =0. Take the curl: VXE 4+ x(v x B) =0. But By Oe wx 8) aed BH Faraday’s law says Vx = (b) V-B= 0% §B-da= 5,84 and R: ot {or any closed surface. Apply this at time (t+ dt) to the surface consisting of Beer dda f Dieta da [Bee + a -da =o In Is (the sign change in the third term comes from switching outward da to inward da). db= Bet d)-da- [ De-da= B+ at) — BU van ff Bt + dt) R) HO Iacoade + sind], (7 > R) Trot? (orhete m = nM); = eo(B > B) = = ta SBF x B)sind, and (#6) = 6,50 rx p= Ho single x bare e= Me anata x8) sin? (P?sin8 dr d0a) [arm wou a4 3 avon (8) (3)- ewer) (b) Apply Faraday’s law to the ring shown: @ 2 (2, aM fe A= Be2ar sind) =~ = —a(r sind) (dott) A > Ho dM (rsin8) 6. poo dM. (=% ‘The free ona patch of surface (dn) ie dP = oR da = - 24H (rsind)dad (= =a) “The torque on the patch is dN = r x dF = meat ("2 sind) da(e x 3). But (@ x 3) = ~6, and we want only the z component (6, = ~ sind): nae [2a (+? sind dO d9) =-meiM an(§ ) ny = ifaw 7 Manis (same as (a) 151 is ait) The charge below © ake (c) Let the charge on the sphere at time ¢ be q(t); the charge density is o = r (“south of") the ring in the figure is 2 yoy = sa 1p = 0 (2k ) [sing a = 5 (—cosd)Ig = F(1 + e038). pa + c036), and hence So the total current crossing the ring (Rowing “north”) is I(t) 1_dg (1+ 0086) BACs") §. The force on 2 patch of area da is dF = (KK x B)da. 1 Ki) = a and ny {2PM ; Ho §2ROM 09 ose + sin88) [24 + 2cos0# +sin8d); 6 2 oars [ucara + fader Baw = K x B= Atwell Os 2088) 96g x) + 2e080(0 x) an = Rexar= 0M (¥) GHD Fx (dx 8) —cosa(tx di] Rising ad de G(e-2) — 208-0) = aM (FB) ar omeyrtfcosed + ose danas Me (Gf) a reostrenoanass, ‘The 2 and y components integrate to zero; (6). = — sind, s0 (using f dg = 2n) ~ 3 dg ‘sin? 1@\ |" N= we ( ) 2m) fr + cose)cosd sins = pe oe) MoM RE (da) (2) _ uo yy pada = 8S" (8) G) = pers ‘Therefore 2H 4 RQ | (same as (a) us fra (Lused the average field at the discontinuity—which is the correct thing to do—but in this case you'd get the same answer using either the inside field or the autside field.) Problem 8.9 a . (0) 6 =—9, 2 rab, B= ponlys €= TR. So a at, wy fe d= ~S & Ena) = -pona?n Ft + B= 152 CHAPTER 8. CONSERVATION LAWS Cc Power: fs are fesiory ten font [ let f Lia The integral is <= —["_ = 2 - (-4) =2 a BVA TE B ( 5) # ante) p,m (peje EAR. ao = = (svootnit) 1 = (Rice = HR. ged Problem 8.10 According to Eqs. 3.104, 4.14, 5.87, and 6.16, the fields are 1 2 = ( Rh, (alm AF -m), (> A), = (4/3)eR°P, and m = (4/9)1RM. Now p= {(B x B)dr, and there are two contetbutions, one from inside the sphere and one from outside. Inside 1 2 8 eho -« | (-+r)» § yx RO(M x Po I( P) ( Swox R9(M x P) - Outside: bs Pan ogee te Now Fx(pxm) = p(-m)~m(#-p),s07 p)~(é-p)(F xm), whereas using the BAC- CAB rule directly gives # x{# x (px m)] = #[#-(pxm)]=-(pxm)(F-E). So {(3(p-#) F ~ p] x [3(m-#)# — m]} = ~3(p-F)(¢xm) +3(m-F)(F xp) +(pxm) = 3 {FF -(p x m)) - (p x m)}+(p=m) = ~2(pxm) +3[F-(pxm)] Poor = B25 f 4 {-2(p x m) + 94fF (px m)]) sin Bara ae, ‘To evaluate the integral, set the z axis along (p x m); then #-(p x m) = |p x m|cos@. Meanwhile, sin oos-+sinBsin 7 -rc0808. But sing and os¢ integrate to er, 80 the X and 9 terns drop out, leaving por = hts ([” Sar) {-2tp xm fsinoaoce-+ ain «sia [cot esineav a} = (-#)/7 [20m myers te my E] = al xm) =~ hip (Jeer) « (Gee) = Brox) “Tae R°(M x P) 4 na = (S48) nttoree) 153 Problem 8.11 (a) From Eq, 5.68 and Prob. 5.36, aR Ho 2 2cos8# + sind), with m= drow rR:B ‘The energy stored in the electric field is (Ex. 2.8) Le 8néo R ‘The energy density of the internal magnetic field is: un = geet = 5h (JutogSg) = BEES, 0 Wp, = MEE fat = WEE ‘The energy density inthe extemal magnetic fl 1B om upg 1 ° (bcos? 0+ sin?) = 2h 16x? 7 Ta(aeay ve (Bese + 1), 50 poe?u?R 108% We. = aa [3 veh (ewes ysinsan fp = RS Fs) (12 We = Wan + Mou W =WetWo= ( Same ae Ped. 888), with Q- and m+ Bevis OER Oxh (9)(7)(1.05 x 10-4) Fale Fe” Texto TM = BBX Wa] [i 22) ] me: [td (By ] 1d CRAP) nos 3x Z (2.01 x 105(L6 x 10-19)? 9.23.x 10-10 = R= FER x 0-011 x 10S RIOE SIT = [Bld x 10 rad/s] Since wR, the speed of a point on the equator, is 300 times the speed of light, this “classical” model is clearly unrealistic. Problem 8.12 Beg R "36x 2.95 x 10" m; ate rae flotm # _ uotm __(e = d2) ar 7 de GEE E = Srdcos dy 14 CHAPTER 8. CONSERVATION LAWS Momentum density (Bq, 8.33): B= OE XD) = any a+ arden Angular momentum density (Eq. 8.34) ‘The x and y components will integrate to zero; using (#). = cos4, we have: _Hotetnd 5 +?(cos? @ — 1), Gy) aera a [Tee r?sin@drdedd. Let w= cosd: cosy Hodedmd co 2(2% Do the r integral fist: Gon Tawa ~ ga w a=) ii rar rua) { P¥e— may” WATE ‘Then Hod 1+ udu = Modem Ce or Problem 8.13 (a) The rotating shell at radios 8 produces solencdal magnetic eis B= poK 2, where K = oyuyt, and 06 = ~325. $0 B= MME g (a <5 ws) %, (8 sdsdg; (2-0)? +y"]= atest s (scosg ~a) : Ws Beacosay 27 cospde on A an de = vo | (AF Boosd) BO- ) f GtBad * Here A? — B® = (5? + a2)? — 4s%a? = s +2870? +a — 48a? = (5? —02)?, VA? — BP =a? — 3? - manly [f.- (£28) « Joo mrts [ ads =| POLE ? +a? — 2sacos¢. 2a

También podría gustarte

  • Electrodinamica Notas
    Electrodinamica Notas
    Documento409 páginas
    Electrodinamica Notas
    cesarin1283
    Aún no hay calificaciones
  • Jovenes Preguntan
    Jovenes Preguntan
    Documento8 páginas
    Jovenes Preguntan
    Fabio Hernández Hernández
    Aún no hay calificaciones
  • Parcial 1 2013
    Parcial 1 2013
    Documento1 página
    Parcial 1 2013
    Fabio Hernández Hernández
    Aún no hay calificaciones
  • Presentación 1
    Presentación 1
    Documento3 páginas
    Presentación 1
    Fabio Hernández Hernández
    Aún no hay calificaciones
  • Examen Final Mecanica de Fluidos
    Examen Final Mecanica de Fluidos
    Documento2 páginas
    Examen Final Mecanica de Fluidos
    Fabio Hernández Hernández
    Aún no hay calificaciones
  • Boletin 1
    Boletin 1
    Documento3 páginas
    Boletin 1
    Fabio Hernández Hernández
    Aún no hay calificaciones
  • Canales Abiertos
    Canales Abiertos
    Documento8 páginas
    Canales Abiertos
    Fundacion Sire
    Aún no hay calificaciones
  • Taller 2
    Taller 2
    Documento2 páginas
    Taller 2
    reospinal
    Aún no hay calificaciones
  • Boletin 1
    Boletin 1
    Documento3 páginas
    Boletin 1
    Fabio Hernández Hernández
    Aún no hay calificaciones
  • V56n1a1 PDF
    V56n1a1 PDF
    Documento7 páginas
    V56n1a1 PDF
    Fabio Hernández Hernández
    Aún no hay calificaciones
  • Tecnicas de Carac Presentacion
    Tecnicas de Carac Presentacion
    Documento5 páginas
    Tecnicas de Carac Presentacion
    Fabio Hernández Hernández
    Aún no hay calificaciones
  • Expos Ici PN de Motivos Def
    Expos Ici PN de Motivos Def
    Documento71 páginas
    Expos Ici PN de Motivos Def
    Marcos Polo
    Aún no hay calificaciones
  • LaTeX1 PDF
    LaTeX1 PDF
    Documento7 páginas
    LaTeX1 PDF
    Fabio Hernández Hernández
    Aún no hay calificaciones
  • Tarea 2
    Tarea 2
    Documento1 página
    Tarea 2
    Fabio Hernández Hernández
    Aún no hay calificaciones
  • Tarea 1
    Tarea 1
    Documento1 página
    Tarea 1
    Fabio Hernández Hernández
    Aún no hay calificaciones